SlideShare a Scribd company logo
1 of 49
Download to read offline
500
Bài Toán B t ð ng Th c Ch n L c
           Cao Minh Quang
                  ♦♦♦♦♦




      Vĩnh Long, Xuân M u Tý, 2008
500 Bài Toán B t ð ng Th c Ch n L c                                                            Cao Minh Quang


             500 Bài Toán B t ð ng Th c Ch n L c
                                                      ♦♦♦♦♦
1. Cho a, b, c là các s th c dương. Ch ng minh r ng

                                     2                       2                    2    3 2
                      a 2 + (1− b) + b 2 + (1− c) + c 2 + (1− a ) ≥                        .
                                                                                        2
                                                      Komal
2. [ Dinu Serbănescu ] Cho a, b, c ∈ (0,1) . Ch ng minh r ng

                                     abc + (1− a )(1− b)(1− c) < 1 .

                                     Junior TST 2002, Romania
3. [ Mircea Lascu ] Cho a, b, c là các s th c dương th a mãn ñi u ki n abc = 1 . Ch ng
minh r ng
                           b+c c +a a +b
                              +     +    ≥ a + b + c + 3.
                            a     b   c
                                          Gazeta Matematică
4. N u phương trình x 4 + ax3 + 2 x 2 + bx + 1 = 0 có ít nh t m t nghi m th c, thì
                                                 a 2 + b2 ≥ 8 .
                                 Tournament of the Towns, 1993
5. Cho các s th c x, y, z th a mãn ñi u ki n x 2 + y 2 + z 2 = 1 . Hãy tìm giá tr l n nh t c a
bi u th c
                                           x3 + y 3 + z 3 − 3xyz .
6. Cho a, b, c, x, y, z là các s th c dương th a mãn ñi u ki n x + y + z = 1 . Ch ng minh
r ng
                   ax + by + cz + 2 ( xy + yz + zx )(ab + bc + ca ) ≤ a + b + c .

                                                Ukraine, 2001
7. [ Darij Grinberg] Cho a, b, c là các s th c dương. Ch ng minh r ng
                             a              b                c                 9
                                     +                +              ≥                 .
                         (b + c)
                                 2
                                         (c + a )
                                                  2              2
                                                          ( a + b)       4 (a + b + c)

8. [ Hojoo Lee ] Cho a, b, c ≥ 0 . Ch ng minh r ng

    a4 + a2b2 + b4 + b4 + b2c2 + c4 + c4 + c2a2 + a4 ≥ a 2a2 + bc + b 2b2 + ca + c 2c2 + ab .
                                          Gazeta Matematică
9. Cho a, b, c là các s th c dương th a mãn ñi u ki n abc = 2 . Ch ng minh r ng

                          a 3 + b 3 + c3 ≥ a b + c + b c + a + c a + b .
                                         JBMO 2002 Shortlist
10. [ Ioan Tomescu ] Cho x, y, z là các s th c dương. Ch ng minh r ng
                                             xyz                     1
                                                                   ≤ 4.
                             (1 + 3x)( x + 8 y )( y + 9 z )( z + 6) 7

2
500 Bài Toán B t ð ng Th c Ch n L c                                                 Cao Minh Quang

                                        Gazeta Matematică
11. [ Mihai Piticari, Dan Popescu ] Cho a, b, c là các s th c dương th a mãn ñi u ki n
a + b + c = 1 . Ch ng minh r ng
                              5 (a 2 + b 2 + c 2 ) ≤ 6 (a 3 + b 3 + c3 ) +1 .

12. [ Mircea Lascu ] Cho x1 , x2 ,..., xn ∈ ℝ , n ≥ 2, a > 0 sao cho

                                                                            a2
                         x1 + x2 + ... + xn = a, x12 + x2 + ... + xn ≤
                                                        2          2
                                                                                .
                                                                           n −1
    Ch ng minh r ng
                                            2a 
                                      xi ∈ 0,  , i = 1, 2,..., n .
                                            n 
13. [ Adrian Zahariuc ] Cho a, b, c ∈ (0,1) . Ch ng minh r ng

                           b a        c b        a c
                                  +          +           ≥1 .
                        4b c − c a 4c a − a b 4a b − b c
14. Cho a, b, c là các s th c dương th a mãn ñi u ki n abc ≤ 1 . Ch ng minh r ng
                                       a b c
                                        + + ≥ a +b+c .
                                       b c a
15. [ Vasile Cirtoaje, Mircea Lascu ] Cho a, b, c, x, y, z là các s th c dương th a mãn ñi u
ki n a + x ≥ b + y ≥ c + z , a + b + c = x + y + z . Ch ng minh r ng
                                         ay + bx ≥ ac + xz .
16. [ Vasile Cirtoaje, Mircea Lascu ] Cho a, b, c là các s th c dương th a mãn ñi u ki n
abc = 1 . Ch ng minh r ng
                                           3          6
                                  1+            ≥             .
                                       a + b + c ab + bc + ca
                                   Junior TST 2003, Romania
17. Cho a, b, c là các s th c dương. Ch ng minh r ng
                                  a 3 b3 c 3 a 2 b 2 c 2
                                     + + ≥ + + .
                                  b2 c2 a 2  b   c   a
                                       JBMO 2002 Shortlist
18. Cho x1 , x2 ,..., xn > 0, n > 3 th a mãn ñi u ki n x1 x2 ...xn = 1 . Ch ng minh r ng
                               1           1                    1
                                       +          + ... +                >1.
                         1 + x1 + x1 x2 1 + x2 x3         1 + xn + xn x1
                                            Russia, 2004
19. [ Marian Tetiva ] Cho x, y, z là các s th c dương th a ñi u ki n x 2 + y 2 + z 2 + 2 xyz = 1 .
Ch ng minh r ng
           1
   a) xyz ≤ ,
           8
                 3
   b) x + y + z ≤ ,
                 2

                                                                                                     3
500 Bài Toán B t ð ng Th c Ch n L c                                                Cao Minh Quang

                     3
    c) xy + yz + zx ≤ ≤ x 2 + y 2 + z 2 ,
                     4
                     1
    d) xy + yz + zx ≤ + 2 xyz .
                     2
20. [ Marius Olteanu ] Cho x1 , x2 ,..., x5 ∈ ℝ sao cho x1 + x2 + ... + x5 = 0 . Ch ng minh r ng

                                 cos x1 + cos x2 + ... + cos x5 ≥ 1 .
                                       Gazeta Matematică
21. [ Florina Cârlan, Marian Tetiva ] Cho x, y, z là các s th c dương th a mãn ñi u ki n
x + y + z = xyz . Ch ng minh r ng

                         xy + yz + zx ≥ 3 + x 2 + 1 + y 2 + 1 + z 2 + 1 .
22. [ Laurentiu Panaitopol ] Cho x, y, z là các s th c th a mãn ñi u ki n x, y , z > −1 .
Ch ng minh r ng
                              1+ x2      1+ y2     1+ z 2
                                       +         +           ≥2.
                             1+ y + z 2 1+ z + x2 1+ x + y 2
                                              JBMO, 2003
23. Cho a, b, c là các s th c dương th a mãn ñi u ki n a + b + c = 1 . Ch ng minh r ng
                                  a2 + b b2 + c c2 + a
                                        +      +       ≥ 2.
                                  b+c     c+a    a +b
24. Cho a, b, c ≥ 0 th a mãn ñi u ki n a 4 + b 4 + c 4 ≤ 2 (a 2b 2 + b 2 c 2 + c 2 a 2 ) . Ch ng minh
r ng
                                 a 2 + b 2 + c 2 ≤ 2 (ab + bc + ca ) .
                                              Kvant, 1988
25. Cho x1 , x2 ,..., xn > 0, n > 2 th a mãn ñi u ki n
                             1        1                1      1
                                 +         + ... +         =     .
                         x1 +1998 x2 +1998         xn +1998 1998
    Ch ng minh r ng
                                              n   x1 x2 ...xn
                                                              ≥ 1998 .
                                                   n −1
                                                  Vietnam, 1998
26. [Marian Tetiva ] Cho x, y, z là các s th c dương th a mãn ñi u ki n x 2 + y 2 + z 2 = xyz .
Ch ng minh r ng
    a) xyz ≥ 27,
    b) xy + yz + zx ≥ 27 ,
    c) x + y + z ≥ 9 ,
    d) xy + yz + zx ≥ 2 ( x + y + z ) + 9 .
27. Cho x, y, z là các s th c dương th a mãn ñi u ki n x + y + z = 3 . Ch ng minh r ng

                                    x + y + z ≥ xy + yz + zx .

4
500 Bài Toán B t ð ng Th c Ch n L c                                                   Cao Minh Quang

                                              Russia 2002
28. [ D. Olteanu ] Cho a, b, c là các s th c dương. Ch ng minh r ng
                 a+b       a       b+c      b       c+a      c      3
                      .          +     .          +     .          ≥ .
                 b + c 2a + b + c c + a 2b + c + a a + b 2c + a + b 4
                                         Gazeta Matematică
29. Cho a, b, c là các s th c dương. Ch ng minh r ng
                                a b c c +a a+b b+c
                                 + + ≥    +    +    .
                                b c a c +b a +c b+a
                                               India, 2002
30. Cho a, b, c là các s th c dương. Ch ng minh r ng
                     a3            b3            c3         3(ab + bc + ca)
                  2         2
                              + 2         2
                                            + 2         2
                                                          ≥                 .
                 b − bc + c    c − ac + a    a − ab + b        a +b +c
                      Proposed for the Balkan Mathematical Olympical
31. [ Adrian Zahariuc ] Cho x1 , x2 ,..., xn là các s nguyên ñôi m t phân bi t nhau. Ch ng
minh r ng
                         x12 + x2 + ... + xn ≥ x1 x2 + x2 x3 ... + xn x1 + 2n − 3 .
                                2          2


32. [ Murray Klamkin ] Cho x1 , x2 ,..., xn ≥ 0, n > 2 th a mãn ñi u ki n x1 + x2 + ... + xn = 1 .
Hãy tìm giá tr l n nh t c a bi u th c
                                   x12 x2 + x2 x3 + ... + xn−1 xn + xn x1 .
                                             2             2         2


                                      Crux Mathematicorum
33. Cho x1 , x2 ,..., xn > 0 th a mãn ñi u ki n xk +1 ≥ x1 + x2 + ... + xk v i m i k. Hãy tìm giá tr
l n nh t c a h ng s c sao cho x1 + x2 + ... + xn ≤ c x1 + x2 + ... + xn .
                                         IMO Shortlist, 1986
34. Cho các s th c dương a, b, c, x, y, z th a mãn ñi u ki n a + x = b + y = c + z = 1. Ch ng
minh r ng
                                                 1   1  1 
                                              
                                  (abc + xyz )      + + ≥ 3.
                                                           
                                                            
                                                  ay bz cx 
                                              Russia, 2002
35. [ Viorel Vâjâitu, Alexvàru Zaharescu ] Cho a, b, c là các s th c dương. Ch ng minh
r ng
                          ab         bc         ca     1
                                +          +          ≤ (a + b + c) .
                      a + b + 2c b + c + 2a c + a + 2b 4
                                         Gazeta Matematică
36. Cho a, b, c, d là các s th c th a mãn ñi u ki n a 2 + b 2 + c 2 + d 2 = 1 . Tìm giá tr nh
nh t c a bi u th c
                 a 3 (b + c + d ) + b3 (c + d + a) + c 3 (d + a + b) + d 3 (a + b + c) .
37. [ Walther Janous ] Cho x, y, z là các s th c dương. Ch ng minh r ng



                                                                                                       5
500 Bài Toán B t ð ng Th c Ch n L c                                                              Cao Minh Quang

                          x                            y                            z
                                         +                            +                           ≤1 .
                x + ( x + y )( x + z )       y + ( y + z )( y + x )       z + ( z + x)( z + y )
                                             Crux Mathematicorum
38. Cho a1 , a2 ,..., an , n ≥ 2 là n s th c sao cho a1 < a2 < ... < an . Ch ng minh r ng

                              a1a2 + a2 a3 + ... + an a14 ≥ a2 a14 + a3a2 + ... + a1an .
                                 4       4                              4            4


39. [ Mircea Lascu ] Cho a, b, c là các s th c dương. Ch ng minh r ng
                           b+c c +a a +b     a      b     c .
                              +    +     ≥ 4
                                                 +     +     
                                                              
                            a    b    c     b + c c + a a + b
                                            
40. Cho a1 , a2 ,..., an là các s nguyên dương l n hơn 1. T n t i ít nh t m t trong các s
                                                           3
a1
     a1 , a2 a3 ,..., an−1 an , an a1 nh hơn ho c b ng         3.
                                  Adapted after a well – known problem
41. [ Mircea Lascu, Marian Tetiva ] Cho x, y, z là các s th c dương th a mãn ñi u ki n
xy + yz + zx + 2 xyz = 1 . Ch ng minh r ng
              1
      a) xyz ≤ ,
              8
                    3
      b) x + y + z ≥ ,
                    2
           1 1 1
      c)    + + ≥ 4( x + y + z) ,
           x y z
                                                      2
         1 1 1                  (2 z −1)
      d)  + + − 4( x + y + z) ≥            , z = max { x, y, z } .
         x y z                  z (2 z +1)
42. [ Manlio Marangelli ] Cho x, y, z là các s th c dương. Ch ng minh r ng

                        3( x 2 y + y 2 z + z 2 x )( xy 2 + yz 2 + zx 2 ) ≥ xyz ( x + y + z ) .
                                                                                           3



43. [ Gabriel Dospinescu ] Cho a, b, c là các s th c dương th a mãn ñi u ki n
                                         max {a, b, c} − min {a, b, c} ≤ 1
      Ch ng minh r ng
                               1 + a 3 + b3 + c 3 + 6abc ≥ 3a 2b + 3b 2 c + 3c 2 a .
44. [ Gabriel Dospinescu ] Cho a, b, c là các s th c dương. Ch ng minh r ng
                              a 2  b2  c2                1 1 1
                      27 + 2 + 2 + 2 +  ≥ 6 (a + b + c ) + +  .
                                          
                           
                           
                                   
                                   
                               bc      
                                         
                                      ca     
                                               
                                            ab               a b c
                                                              
                                                                    
                                                                     

            1              a2
45. Cho a0 = , a k+1 = ak + k . Ch ng minh r ng
            2               n
                                                    1
                                                  1− < an < 1 .
                                                    n
                                                 TST Singapore
46. [ Călin Popa ] Cho a, b, c ∈ (0,1) th a mãn ñi u ki n ab + bc + ca = 1 . Ch ng minh r ng


6
500 Bài Toán B t ð ng Th c Ch n L c                                                              Cao Minh Quang

                            a      b      c    3 1− a 2 1− b2 1− c 2 
                                                                      
                                +      +      ≥       +     +       .
                         1 − a 2 1− b 2 1− c 2 4  a
                                                          b     c   
                                                                      
47. [ Titu Vàreescu, Gabriel Dospinescu ] Cho x, y, z ≤ 1 th a mãn ñi u ki n x + y + z = 1 .
Ch ng minh r ng
                                         1        1        1     27
                                            2
                                              +      2
                                                       +      2
                                                                ≤ .
                                       1+ x     1+ y     1+ z    10
48. [ Gabriel Dospinescu ] Cho           x + y + z = 1 . Ch ng minh r ng
                               2         2           2
                        (1− x) (1− y ) (1− z ) ≥ 215 xyz ( x + y )( y + z )( z + x) .
49. Cho x, y, z là các s th c dương th a mãn ñi u ki n xyz = x + y + z +2 . Ch ng minh r ng
     a) xy + yz + zx ≥ 2 ( x + y + z ) ,
                              3
     b)        x+ y+ z≤         xyz .
                              2
50. Cho x, y, z là các s th c th a mãn ñi u ki n x 2 + y 2 + z 2 = 2 . Ch ng minh r ng
                                                 x + y + z ≤ xyz + 2 .
                                              IMO Shortlist, 1987
51. [ Titu Vàreescu, Gabriel Dospinescu ] Cho x1 , x2 ,..., xn ∈ (0,1) và σ là m t hoán v c a
{1, 2,..., n} . Ch ng minh r ng
                                                 n   
                                             ∑ xi   n
                                            
                                            
                                                      
                                                                     
                                    n
                                       1    1 + i=1 .
                                                            1       
                                                                      
                                   ∑ 1− x ≥
                                                      ∑
                                                        1− x . x  .
                                                        
                                                    n   i=1         
                                                                      
                                   i=1                      i σ(i ) 
                                         i
                                            
                                                     
                                                      
                                                     
                                                                              n
                                                                                    1
52. Cho x1 , x2 ,..., xn là các s th c dương th a mãn ñi u ki n               ∑ 1+ x        = 1 . Ch ng minh r ng
                                                                              i=1       i

                                             n                   n
                                                                       1
                                         ∑         xi ≥ (n −1) ∑
                                                                       xi
                                                                          .
                                             i=1                 i=1

                                                   Vojtech Jarnik
                                                                                                        n
53. [ Titu Vàreescu ] Cho n > 3 và a1 , a2 ,..., an là các s th c th a mãn ñi u ki n                   ∑a ≥ n i
                                                                                                        i=1
     n
và   ∑a    2
           i   ≥ n 2 . Ch ng minh r ng
     i=1

                                             max {a1 , a2 ,..., an } ≥ 2 .
                                                   USAMO, 1999
54. [ Vasile Cirtoaje ] Cho a, b, c, d là các s th c dương. Ch ng minh r ng
                                     a −b b−c c − d d −a
                                         +   +     +     ≥0.
                                     b+c c +d d +a a +b
55. Cho x, y là các s th c dương. Ch ng minh r ng
                                                     x y + yx >1 .
                                                                                                                  7
500 Bài Toán B t ð ng Th c Ch n L c                                                              Cao Minh Quang

                                                   France, 1996
56. Cho a, b, c là các s th c dương th a mãn ñi u ki n abc = 1 . Ch ng minh r ng
                                 (a + b)(b + c)(c + a ) ≥ 4 (a + b + c −1) .
                                                   MOSP, 2001
57. Cho a, b, c là các s th c dương. Ch ng minh r ng

               (a 2 + b2 + c2 )(a + b − c)(b + c − a)(c + a − b) ≤ abc (ab + bc + ca) .
58. [ D.P.Mavlo ] Cho a, b, c là các s th c dương. Ch ng minh r ng
                              1 1 1 a b c    (a + 1)(b +1)(c +1)
                 3+ a +b + c + + + + + + ≥ 3                     .
                              a b c b c a          1 + abc
                                                   Kvant, 1988
59. [ Gabriel Dospinescu ] Cho x1 , x2 ,..., xn là các s                     th c dương th a mãn ñi u ki n
x1 x2 ...xn = 1 . Ch ng minh r ng

                                                     n       1
                                                                                     n
                                          n                 n
                                     n .∏( x + 1) ≥ ∑ xi + ∑  .
                                      n       n     
                                                    
                                                               
                                                               
                                        i=1
                                              i
                                                          i =1
                                                               
                                                              x       i=1   i

60. Cho a, b, c, d là các s th c dương th a mãn ñi u ki n a + b + c = 1 . Ch ng minh r ng
                                                            1 1 d 
                                                                    
                                 a 3 + b3 + c3 + abcd ≥ min  , +  .
                                                             4 9 27 
                                                            
                                                                    
                                                                     
                                                   Kvant, 1993
61. Cho a, b, c là các s th c dương. Ch ng minh r ng

    ∑ (1+ a ) (1 + b ) (a − c) (b − c) ≥ (1 + a )(1 + b )(1 + c )(a − b) (b − c) (c − a) .
              2 2        2 2          2        2            2          2                 2   2      2      2



                                                      AMM
62. [ Titu Vàreescu, Mircea Lascu ] Cho x, y, z là các s th c dương th a mãn ñi u ki n
xyz = 1 và α ≥ 1. Ch ng minh r ng
                                           xα   yα   zα  3
                                              +    +    ≥ .
                                          y+z z+x x+ y 2
63. Cho x1, x2 ,..., xn , y1, y2 ,..., yn ∈ ℝ th a mãn ñi u ki n x12 + x2 +... + xn = y12 + y2 +... + yn =1 .
                                                                        2         2          2         2


Ch ng minh r ng
                                                                  n             
                                      ( x1 y2 − x2 y1 ) ≤ 2 1− ∑ xi yi  .
                                                            
                                                       2
                                                                        
                                                                        
                                                             i=1
                                                                       
                                                                        
                                                   Korea, 2001
64. [ Laurentiu Panaitopol ] Cho a1 , a2 ,..., an là các s nguyên dương khác nhau t ng ñôi m t.
Ch ng minh r ng
                                                       2n + 1
                               a12 + a2 + ... + an ≥
                                      2          2
                                                              (a1 + a2 + ... + an ) .
                                                         3
                                                  TST Romania
65. [ Călin Popa ] Cho a, b, c là các s th c dương th a mãn ñi u ki n a + b + c = 1 . Ch ng
minh r ng

8
500 Bài Toán B t ð ng Th c Ch n L c                                                                          Cao Minh Quang

                              b c                        c a                       a b               3 3
                                            +                          +                         ≥       .
                     a   (   3c + ab    )       b   (    3a + bc   )       c   (   3b + ca   )        4

66. [ Titu Vàreescu, Gabriel Dospinescu ] Cho a, b, c, d là các s th c th a mãn ñi u ki n
(1 + a 2 )(1+ b2 )(1+ c 2 )(1 + d 2 ) = 16 . Ch           ng minh r ng

                              −3 ≤ ab + bc + cd + da + ac + bd − abcd ≤ 5 .
67. Cho a, b, c là các s th c dương. Ch ng minh r ng

                               (a 2 + 2)(b2 + 2)(c2 + 2) ≥ 9(ab + bc + ca) .
                                                         APMO, 2004
68. [ Vasile Cirtoale ] Cho x, y, z là các s th c th a mãn các ñi u ki n 0 < x ≤ y ≤ z,
 x + y + z = xyz + 2 . Ch ng minh r ng
    a) (1− xy )(1− yz )(1− zx) ≥ 0 ,
                              32
    b) x 2 y ≤ 1, x 3 y 2 ≤      .
                              27
69. [ Titu Vàreescu ] Cho a, b, c là các s th c dương th a mãn ñi u ki n a + b + c ≥ abc .
Ch ng minh r ng ít nh t m t trong ba b t ñ ng th c sau ñây là ñúng
                               2 3 6    2 3 6    2 3 6
                                + + ≥ 6, + + ≥ 6, + + ≥ 6 .
                               a b c    b c a    c a b
                                                        TST 2001, USA
70. [ Gabriel Dospinescu, Marian Tetiva ] Cho x, y, z là các s th c dương th a mãn ñi u
ki n x + y + z = xyz . Ch ng minh r ng

                                       ( x −1)( y −1)( z −1) ≤ 6 3 −10 .
71. [ Marian Tetiva ] Cho a, b, c là các s th c dương. Ch ng minh r ng
                                                                               2         2               2
                    a3 − b3 b3 − c3 c 3 − a3 (a − b) + (b − c ) + (c − a )
                           +       +        ≤                              .
                     a +b    b+c     c+a                  4
                                                Moldova TST, 2004
72. [ Titu Vàreescu ] Cho a, b, c là các s th c dương. Ch ng minh r ng

                         (a5 − a 2 + 3)(b5 − b2 + 3)(c5 − c 2 + 3) ≥ (a + b + c)3 .
                                                        USAMO, 2004
73. [ Gabriel Dospinescu ] Cho x1 , x2 ,..., xn > 0, n > 2 th a mãn ñi u ki n
                                             n     n 1    
                                             x 
                                            ∑ k   ∑  = n 2 + 1 .
                                                              
                                             k =1  k =1 xk 
                                                           

Ch ng minh r ng
                                      n 2  n 1 
                                                               2
                                      x 
                                      ∑ k  ∑ 2  > n + 4 +
                                                        2
                                                                       .
                                                      
                                      k =1  k =1 xk 
                                                             n (n −1)
                                           
74. [ Gabriel Dospinescu, Mircea Lascu, Marian Tetiva ] Cho a, b, c là các s th c dương.
Ch ng minh r ng

                                                                                                                              9
500 Bài Toán B t ð ng Th c Ch n L c                                                                          Cao Minh Quang

                               a 2 + b 2 + c 2 + 2abc + 3 ≥ (1 + a)(1 + b)(1 + c) .
75. [ Titu Vàreescu, Zuming Feng ] Cho a, b, c là các s th c dương. Ch ng minh r ng
                                           2                       2                          2
                          ( 2a + b + c )    (2b + a + c)    (2c + b + c)
                                         2
                                           + 2           2
                                                           + 2           2
                                                                           ≤8.
                          2a + (b + c)
                             2
                                            2b + (a + c)    2c + (a + b)
                                                      USAMO, 2003
76. Cho x, y là các s th c dương và m, n là các s nguyên dương. Ch ng minh r ng

        (n −1)(m −1)( x m+n + y m+n ) + (m + n −1)( x m y n + x n y m ) ≥ mn ( x m+n−1 y + y m+n−1 x) .
                                   Austrian – Polish Competition, 1995
77. Cho a, b, c, d , e là các s th c dương th a mãn ñi u ki n abcde = 1 . Ch ng minh r ng
         a + abc       b + bcd       c + cde       d + dea       e + eab     10
                   +             +             +             +              ≥ .
      1 + ab + abcd 1 + bc + bcde 1 + cd + cdea 1 + de + deab 1 + ea + eabc   3
                                               Crux Mathematicorum
                                     π
78. [ Titu Vàreescu ] Cho a, b, c ∈ 0,  . Ch ng minh r ng
                                     2
                                    
                                       
                                        
     sin a.sin (a − b).sin (a − c ) sin b.sin (b − c ).sin (b − a ) sin c.sin (c − a ).sin (c − b)
                                   +                               +                               ≥0.
              sin (b + c )                   sin (c + a )                    sin (a + b)
                                                      TST 2003, USA
79. Cho a, b, c là các s th c dương. Ch ng minh r ng

            a 4 + b4 + c 4 + a 2b 2 + b 2c 2 + c 2 a 2 ≥ a 3b + b3c + c 3a + ab3 + bc3 + ca 3 .
                                    KMO Summer Program Test, 2001
80. [ Gabriel Dospinescu, Mircea Lascu ] Cho a1 , a2 ,..., an > 0, n > 2 th a mãn ñi u ki n
a1a2 ...an = 1 . Hãy tìm h ng s kn nh nh t sao cho
                        a1a2                           a2 a3                                an a1
                                       +                               + ... +                               ≤ kn .
               (a2
                 1   + a2 )(a + a1 )
                               2
                               2           (a  2
                                               2    + a3 )(a + a2 )
                                                               2
                                                               3                 (a  2
                                                                                     n   + a1 )(a12 + an )

81. [ Vasile Cirtoaje ] Cho a, b, c, x, y, z là các s th c dương. Ch ng minh r ng
                                                                                 2
              ax + by + cz +       (a 2 + b2 + c 2 )( x 2 + y 2 + z 2 ) ≥ 3 (a + b + c)( x + y + z ) .
                                                       Kvant, 1989
82. [ Vasile Cirtoaje ] Cho a, b, c là ñ dài ba c nh c a m t tam giác. Ch ng minh r ng
                                       a b c       b c a
                                      3 + + −1 ≥ 2  + +  .
                                              
                                                         
                                                           
                                       b c a 
                                                    a b c
                                                     
83. [ Walther Janous ] Cho x1 , x2 ,..., xn > 0, n > 2 th a mãn ñi u ki n x1 + x2 + ... + xn = 1 .
Ch ng minh r ng
                                                          n         
                                                   1 + 1  ≥  n − xi  .
                                                n

                                               ∏ x  ∏ 1 − x 
                                                   
                                               i=1 
                                                          
                                                          
                                                           i=1 
                                                                
                                                                       
                                                                       
                                                                       
                                                           i                i

                                               Crux Mathematicorum


10
500 Bài Toán B t ð ng Th c Ch n L c                                                             Cao Minh Quang

84. [ Vasile Cirtoaje, Gheoghe Eckstein ] Cho x1 , x2 ,..., xn là các s th c dương th a mãn ñi u
ki n x1 x2 ...xn = 1 . Ch ng minh r ng
                             1         1                 1
                                   +          + ... +           ≤1 .
                          n −1 + x1 n −1 + x2         n −1 + xn
                                       TST 1999, Romania
85. [ Titu Vàreescu ] Cho a, b, c là các s th c không âm th a ñi u ki n a2 +b2 +c2 +abc = 4 .
Ch ng minh r ng
                                  0 ≤ ab + bc + ca − abc ≤ 2 .
                                         USAMO, 2001
86. [ Titu Vàreescu ] Cho a, b, c là các s th c dương. Ch ng minh r ng
               a +b +c 3
                                          {(              ) (                  ) (          ) }.
                                                          2                    2            2
                      − abc ≤ max               a− b ,             b− c ,            c− a
                  3
                                         TST 2000, USA
87. [ Kiran Kedlaya ] Cho a, b, c là các s th c dương. Ch ng minh r ng

                           a + ab + 3 abc 3 a + b a + b + c
                                         ≤ a.    .          .
                                 3            2       3
88. Tìm h ng s k l n nh t sao cho v i b t kì s nguyên dương n không chính phương, ta
có

                                      (1+ n ) sin (π n ) > k .
                          Vietnamese IMO Training Camp, 1995
                                                                                                     3
89. [ Tr n Nam Dũng ] Cho x, y, z là các s th c dương th a ñi u ki n ( x + y + z ) = 32 xyz .
Tìm giá tr l n nh t và giá tr nh nh t c a bi u th c
                                              x4 + y4 + z 4
                                                           4
                                                               .
                                              (x + y + z)
                                         Vietnam, 2004
90. [ George Tsintifas ] Cho a, b, c, d là các s th c dương. Ch ng minh r ng
                      3       3        3       3                                  4
               (a + b) (b + c) (c + d ) (d + a) ≥ 16a 2b2 c 2 d 2 (a + b + c + d ) .
                                   Crux Mathematicorum
91. [ Titu Vàreescu, Gabriel Dospinescu ] Cho a, b, c là các s th c không âm th a mãn ñi u
ki n a + b + c = 1 và n là s nguyên dương. Tìm giá tr l n nh t c a bi u th c
                                      (ab)        (bc)         (ca)
                                          n           n            n

                                              +           +            .
                                    1− ab         1− bc       1− ca
92. Cho a, b, c là các s th c dương. Ch ng minh r ng
                          1         1          1               3
                               +          +           ≥                 .
                      a (1 + b) b (1 + c ) c (1 + a )   3
                                                          abc 1 + 3 abc    (          )
93. [Tr n Nam Dũng ] Cho a, b, c là các s th c dương th a mãn ñi u ki n a 2 + b2 + c 2 = 9 .
Ch ng minh r ng

                                                                                                             11
500 Bài Toán B t ð ng Th c Ch n L c                                                      Cao Minh Quang

                                         2 (a + b + c) − abc ≤ 10 .
                                               Vietnam, 2002
94. [ Vasile Cirtoaje ] Cho a, b, c là các s th c dương. Ch ng minh r ng
                                                                
          a + 1 −1b + 1 −1 + b + 1 −1c + 1 −1 + c + 1 −1a + 1 −1 ≥ 3 .
                                                                
          
              b          
                         c          c          
                                                a          a     b  

95. [ Gabriel Dospinescu ] Cho n là s nguyên l n hơn 2. Tìm s th c l n nh t mn và s
th c nh nh t M n sao cho v i các s th c dương b t kì x1 , x2 ,..., xn (xem xn = x0 , xn+1 = x1 ),
ta có
                                         n
                                                         xi
                               mn ≤ ∑                                   ≤ Mn .
                                      i=1    xi−1 + 2 (n −1) xi + xi +1
96. [ Vasile Cirtoaje ] Cho x, y, z là các s th c dương. Ch ng minh r ng
                       1             1             1               9
                    2         2
                                + 2         2
                                              + 2         2
                                                            ≥            2
                                                                           .
                   x + xy + y    y + yz + z    z + zx + x     (x + y + z)
                                             Gazeta Matematică
97. [ Vasile Cirtoaje ] Cho a, b, c, d là các s th c dương. Ch ng minh r ng

         2 (a3 +1)(b3 +1)(c 3 + 1)(d 3 +1) ≥ (1 + abcd )(1 + a 2 )(1 + b 2 )(1 + c 2 )(1 + d 2 ) .

                                             Gazeta Matematică
98. Cho a, b, c là các s th c dương. Ch ng minh r ng
                                                                  4 4
                                 4              4
                         (a + b) + (b + c) + (c + a) ≥
                                                              4

                                                                  7
                                                                    (a + b 4 + c 4 ) .
                                             Vietnam TST, 1996
99. Cho a, b, c là các s th c dương th a mãn ñi u ki n abc = 1 . Ch ng minh r ng
                       1        1        1       1     1     1
                            +        +        ≤     +     +      .
                    1+ a + b 1+ b + c 1+ c + a 2 + a 2 + b 2 + c
                                               Bulgaria, 1997
100. [Tr n Nam Dũng ] Cho a, b, c là các s th c dương th a 21ab + 2bc + 8ca ≤ 12 . Tìm
giá tr nh nh t c a bi u th c
                                                    1 2 3
                                                     + + .
                                                    a b c
                                               Vietnam, 2001
101. [ Titu Vàreescu, Gabriel Dospinescu ] Cho a, b, c, x, y, z là các s th c dương th a mãn
ñi u ki n xy + yz + zx = 3 . Ch ng minh r ng
                          a             b               c
                             ( y + z)+     ( z + x) +      ( x + y) ≥ 3 .
                         b+c           c+a            a +b
102. Cho a, b, c là các s th c dương. Ch ng minh r ng
                                     2                    2                   2
                         (b + c − a )   (c + a − b)   (a + b − c)  3
                                2
                                      +        2
                                                    +        2
                                                                  ≥ .
                        (b + c) + a 2 (c + a) + b 2 (a + b) + c 2 5
                                                Japan, 1997
12
500 Bài Toán B t ð ng Th c Ch n L c                                                                   Cao Minh Quang

103. [ Vasile Cirtoaje, Gabriel Dospinescu ] Cho a1 , a2 ,..., an ≥ 0, an = min {a1 , a2 ,..., an } .
Ch ng minh r ng
                                                            a + a2 + ... + an−1      n
                 a1n + a2 + ... + an − na1a2 ...an ≥ (n −1) 1
                        n          n
                                                                                − an  .
                                                                                      
                                                                                      
                                                           
                                                                  n −1               
104. [ Turkervici ] Cho x, y , z , t là các s th c dương. Ch ng minh r ng
                   x 4 + y 4 + z 4 + t 4 + 2 xyzt ≥ x 2 y 2 + y 2 z 2 + z 2t 2 + x 2 z 2 + y 2t 2 .
                                                          Kvant
105. Cho a1 , a2 ,..., an là các s th c dương. Ch ng minh r ng

                                            n 2      n
                                            a ≤
                                           ∑ i 
                                                           ij
                                                  ∑ i + j −1ai a j .
                                            i=1  i , j=1
                                                

106. Cho a1 , a2 ,..., an , b1 , b2 ,..., bn ∈ (1001, 2002) sao cho a12 + a2 + ... + an = b12 + b2 + ... + bn .
                                                                           2          2          2          2


Ch ng minh r ng
                               a13 a2
                                    3
                                           a 3 17
                                  + + ... + n ≤ (a12 + a2 + ... + an ) .
                                                        2          2

                               b1 b2       bn 10
                                                  TST Singapore
107. [ Titu Vàreescu, Gabriel Dospinescu ] Cho a, b, c là các s th c dương th a mãn ñi u
ki n a + b + c = 1 . Ch ng minh r ng

                       (a 2 + b2 )(b2 + c 2 )(c 2 + a 2 ) ≥ 8(a 2b2 + b2c 2 + c 2 a 2 )
                                                                                              2
                                                                                                  .

108. [ Vasile Cirtoaje ] Cho a, b, c, d là các s th c dương th a mãn ñi u ki n abcd = 1 .
Ch ng minh r ng
                                  1               1              1              1
                                       2
                                           +          2
                                                          +          2
                                                                         +          2
                                                                                        ≥1.
                              (1 + a )         (1 + b)        (1 + c)        (1 + d )
                                               Gazeta Matematică
109. [ Vasile Cirtoaje ] Cho a, b, c là các s th c dương. Ch ng minh r ng
                           a2       b2       c2      a    b    c
                         2    2
                                + 2    2
                                         + 2    2
                                                  ≥    +    +     .
                        b +c     c +a     a +b      b+c c +a a +b
                                               Gazeta Matematică
110. [ Gabriel Dospinescu ] Cho n s th c a1 , a2 ,..., an . Ch ng minh r ng
                                                 2
                                           
                                       a ≤                         2

                                             1≤∑ n ( i
                                      ∑ i 
                                                    a + ... + a j ) .
                                      i∈ℕ* 
                                              i≤ j≤


                                               TST 2004, Romania
111. [Tr n Nam Dũng ] Cho x1 , x2 ,..., xn ∈ [−1,1] th a mãn ñi u ki n x1 + x2 + ... + xn = 0 .
                                                                        3    3          3


Tìm giá tr l n nh t c a bi u th c
                                                  x1 + x2 + ... + xn .
112. [ Gabriel Dospinescu, Călin Popa ] Cho n s th c a1 , a2 ,..., an , n ≥ 2 th a mãn ñi u
ki n a1a2 ...an = 1 . Ch ng minh r ng

                                                                                                                   13
500 Bài Toán B t ð ng Th c Ch n L c                                                                             Cao Minh Quang

                                                         2n n
                    a12 + a2 + ... + an − n ≥
                           2          2
                                                              n −1 (a1 + a2 + ... + an − n) .
                                                        n −1
113. [ Vasile Cirtoaje ] Cho a, b, c là các s th c dương. Ch ng minh r ng

                                       2a     2b    2c
                                           +     +     ≤ 3.
                                      a +b   b+c   c+a
                                             Gazeta Matematică
114. Cho x, y, z là các s th c dương. Ch ng minh r ng
                                                   1                     1                          1    9
                        ( xy + yz + zx)                 2
                                                                  +                   2
                                                                                          +            2
                                                                                                         ≥ .
                                          ( x + y )                 ( y + z)                 ( z + x)  4
                                                        Iran, 1996
115. [ Cao Minh Quang ] Cho x1 , x2 ,..., xn là các s th c dương th a mãn ñi u ki n
                                                n

                                              ∏(3x +1) ≤ 2
                                                i=1
                                                              i
                                                                                  n
                                                                                      .

     Ch ng minh r ng
                                                    n
                                                                  1           n
                                                ∑ 6 x +1 ≥ 3 .
                                                 i=1              i

116. [ Suranyi ] Cho a1 , a2 ,..., an là các s th c dương. Ch ng minh r ng

      (n −1)(a1n + a2 + ... + an ) + na1a2 ...an ≥ (a1 + a2 + ... + an )(a1n−1 + a2 −1 + ... + ann−1 ) .
                    n          n                                                  n



                                  Miklos Schweitzer Competition
117. [ Gabriel Dospinescu ] Cho x1 , x2 ,..., xn > 0 th a mãn ñi u ki n x1 x2 ...xn = 1 . Ch ng
minh r ng
                                                                           n

                                       ∑ (x − x ) ≥ ∑ x
                                                                      2                   2
                                                    i             j                       i   −n .
                                     1≤i≤ j≤n                             i =1

                             A generazation of Tukervici’s Inequality
                                                                1
118. [ Vasile Cirtoaje ] Cho a1 , a2 ,..., an <                    và a1 + a2 + ... + an = 1, n > 2 . Tìm giá tr
                                                              n −1
nh nh t c a bi u th c
                                                n
                                                          a1a2 ...an
                                              ∑          1−(n −1) ai
                                                                     .
                                              i=1


119. [ Vasile Cirtoaje ] Cho a1 , a2 ,..., an ∈ [0,1) th a mãn ñi u ki n

                                              a12 + a2 + ... + an
                                                     2          2
                                                                     3
                                    a=                            ≥    .
                                                       n            3
     Ch ng minh r ng
                                 a1     a           a     na
                                    2
                                      + 2 2 + ... + n 2 ≥       .
                                1− a1 1− a2        1− an 1− a 2
120. [ Vasile Cirtoaje, Mircea Lascu ] Cho a, b, c, x, y, z là các s th c dương th a mãn ñi u
ki n

14
500 Bài Toán B t ð ng Th c Ch n L c                                                              Cao Minh Quang

                   (a + b + c)( x + y + z ) = (a 2 + b 2 + c 2 )( x 2 + y 2 + z 2 ) = 4 .
      Ch ng minh r ng
                                                                 1
                                                    abcxyz <       .
                                                                36
121. [ Gabriel Dospinescu ] Cho x1 , x2 ,..., xn > 0, n > 2 th a mãn ñi u ki n x1 x2 ...xn = 1 . Tìm
h ng s kn nh nh t sao cho
                                 1           1                 1
                                        +           + ... +           ≤ n −1 .
                              1 + kn x1   1 + kn x2         1 + kn xn
                                             Mathlinks Contest
122. [ Vasile Cirtoaje, Gabriel Dospinescu ] Cho x1 , x2 ,..., xn > 0, n > 2 th a mãn ñi u ki n
x12 + x2 + ... + xn = 1 . Tìm h ng s kn l n nh t sao cho
       2          2



                                  (1− x1 )(1− x2 )...(1− xn ) ≥ kn x1 x2 ...xn .


123. Cho a, b, c là các s th c dương th a mãn ñi u ki n abc = 1 . Ch ng minh r ng
                                     1           1          1     3
                                            + 3        + 3       ≥ .
                                  a (b + c ) b (c + a ) c (a + b) 2
                                   3


                                                 IMO, 1995
124. Cho a, b, c là các s th c dương th a mãn ñi u ki n abc = 1 . Ch ng minh r ng
                             ab         bc         ca
                                   + 5        + 5           ≤ 1.
                         a + b + ab b + c + bc c + a 5 + ca
                           5  5          5


                                             IMO Shortlist, 1996
125. Cho a, b, c là các s th c dương th a mãn ñi u ki n abc = 1 . Ch ng minh r ng
                              1 + ab 2 1 + bc 2 1 + ca 2     18
                                   3
                                      +     3
                                               +     3
                                                         ≥ 3          .
                                 c        a        b      a + b3 + c3
                                              Hong Kong, 2000
126. Cho a, b, c là các s th c dương th a mãn ñi u ki n abc = 1 . Ch ng minh r ng
                              1                         1                     1            1
                          2
                                         +          2
                                                                +         2
                                                                                       ≤     .
                  (a +1) + b + 1 (b +1) + c + 1 (c +1) + a + 1
                                   2                        2                      2       2

127. Cho a, b, c là các s th c dương th a mãn ñi u ki n abc = 1 . Ch ng minh r ng
                                                              
                                   a −1 + 1 b −1 + 1 c −1 + 1  ≤ 1 .
                                                              
                                   
                                          b 
                                                    c 
                                                               a
                                                 IMO, 2000
128. Cho a, b, c là các s th c dương th a mãn ñi u ki n abc = 1 . Ch ng minh r ng
                             a3             b3             c3        3
                                     +              +               ≥ .
                       (1 + b)(1 + c) (1 + a)(1 + c) (1 + a )(1 + b) 4
                                             IMO Shortlist, 1998
129. Cho a, b, c là các s th c dương th a mãn ñi u ki n a + b + c = 1 . Ch ng minh r ng

                                                                                                              15
500 Bài Toán B t ð ng Th c Ch n L c                                                     Cao Minh Quang

                                       ab   bc   ca  1
                                          +    +    ≤ .
                                      1+ c 1+ a 1+ b 4
130. Cho a, b, c là các s th c dương th a mãn ñi u ki n a + b + c = 1 . Ch ng minh r ng

                                     a 2 + b 2 + c 2 + 2 3abc ≤ 1 .
                                                  Poland, 1999
131. Cho a, b, c là các s th c dương th a mãn ñi u ki n a 2 + b 2 + c 2 = 1 . Ch ng minh r ng
                                                             1
                                         a +b+c +               ≥4 3.
                                                            abc
                                              Macedonia, 1999
132. Cho a, b, c là các s th c dương th a mãn ñi u ki n a + b + c = 1 . Ch ng minh r ng

                     ab + c + bc + a + ca + b ≥ 1 + ab + bc + ca .
133. Cho a, b, c là các s th c dương th a mãn ñi u ki n a + b + c = 1 . Ch ng minh r ng
                         (1 + a)(1 + b)(1 + c) ≥ 8 (1− a )(1− b)(1− c) .
                                                  Russia, 1991
134. Cho a, b là các s th c dương th a mãn ñi u ki n a + b = 1 . Ch ng minh r ng
                                               a2   b2  1
                                                  +    ≥ .
                                              a +1 b +1 3
                                               Hungary, 1996
135. Cho các s th c x, y . Ch ng minh r ng
                                                            2
                                         3( x + y + 1) + 1 ≥ 3 xy .
                                              Columbia, 2001
136. Cho a, b, c là các s th c dương. Ch ng minh r ng

                                               1 1   a   b
                                     2 (a + b) +  ≥ 3 + 3 .
                                              a b
                                              
                                 3
                                                  
                                                      b   a
                                     Czech and Slovakia, 2000
137. Cho a, b, c ≥ 1 . Ch ng minh r ng

                             a −1 + b −1 + c −1 ≤ c (ab + 1) .

                                              Hong Kong, 1998
138. Cho x, y, z là các s th c dương th a mãn ñi u ki n x + y + z = xyz . Ch ng minh r ng
                                     1                  1            1    3
                                              +                 +        ≤ .
                                 1+ x     2
                                                   1+ y     2
                                                                    1+ z  2  2


                                                  Korea, 1998
139. Cho a, b, c là các s th c dương. Ch ng minh r ng
                                 a                      b                    c
                                              +                 +                ≥1 .
                             2                      2                    2
                            a + 8bc                b + 8ca           c + 8ab
                                                  IMO, 2001

16
500 Bài Toán B t ð ng Th c Ch n L c                                             Cao Minh Quang

140. Cho a, b, c, d là các s th c dương. Ch ng minh r ng
                      a           b          c           d      2
                            +           +          +           ≥ .
                 b + 2c + 3d c + 2d + 3a d + a + 3b a + 2b + 3c 3
                                      IMO Shortlist, 1993
141. Cho a, b, c, d là các s th c dương th a mãn ñi u ki n ab + bc + cd + da = 1 . Ch ng
minh r ng
                       a3      b3      c3      d3  1
                           +       +       +      ≥ .
                     b+c +d c +d +a d +a +b a +b+c 3
                                      IMO Shortlist, 1990
142. Cho a, b, c là các s th c dương. Ch ng minh r ng
              a2      b2      c2          bc      ca      ab
            2
                  + 2     + 2      ≥1 ≥ 2     + 2     + 2      .
           a + 2bc b + 2ca c + 2ab     a + 2bc b + 2ca c + 2ab
                                        Romania, 1997
143. Cho a, b, c là các s th c dương. Ch ng minh r ng
                                  a 3 b3 c3
                                     + + ≥ a +b +c .
                                  bc ca ab
                                         Canada, 2002
144. Cho a, b, c là các s th c dương. Ch ng minh r ng
                         1           1           1       1
                     3   3
                               + 3   3
                                           + 3   3
                                                       ≤    .
                    a + b + abc b + c + abc c + a + abc abc
                                          USA, 1997
145. Cho a, b, c là các s th c dương th a mãn ñi u ki n a2 + b2 + c2 = 3 . Ch ng minh r ng
                                    1      1      1   3
                                       +      +      ≥ .
                                 1 + ab 1 + bc 1 + ca 2
                                         Belarus, 1999
146. Cho a, b, c là các s th c dương. Ch ng minh r ng
                                a b c a +b b + c
                                 + + ≥    +      +1.
                                b c a b+c a +b
                                         Belarus, 1998
                    3
147. Cho a, b, c ≥ − , a + b + c = 1 . Ch ng minh r ng
                    4
                                  a      b     c   9
                                  2
                                     + 2   + 2   ≤ .
                                a + 1 b + 1 c + 1 10
                                         Poland, 1996
148. Cho x, y, z là các s th c dương th a mãn ñi u ki n xyz = 1 . Ch ng minh r ng
                        x9 + y 9         y9 + z9           z 9 + x9
                                     + 6               + 6                ≥2.
                    x6 + x3 y 3 + y 6 y + y 3 z 3 + z 6 z + z 3 z 3 + x 6
                                       Roamania, 1997
149. Cho x ≥ y ≥ z > 0 . Ch ng minh r ng

                                                                                             17
500 Bài Toán B t ð ng Th c Ch n L c                                                Cao Minh Quang

                                     x2 y y2 z z 2 x
                                         +    +      ≥ x2 + y2 + z 2 .
                                      z    x     y
                                              Vietnam, 1991
150. Cho a ≥ b ≥ c > 0 . Ch ng minh r ng
                           a 2 − b 2 c 2 − b2 a 2 − c 2
                                    +        +          ≥ 3a − 4b + c .
                               c         a        b
                                               Ukraine, 1992
151. Cho x, y, z là các s th c dương. Ch ng minh r ng

                                 (
                           xyz x + y + z + x 2 + y 2 + z 2        ) ≤ 3+   3
                                                                               .
                            (x   2
                                     + y + z )( xy + yz + zx )
                                        2      2
                                                                       9

                                             Hong Kong, 1997
152. Cho a1 , a2 , ..., an > 0 và a1 + a2 + ... + an < 1 . Ch ng minh r ng
                             a1a2 ...an (1− a1 − a2 − ... − an )         1
                                                                      ≤ n+1 .
                      (a1 + a2 + ... + an )(1− a1 )(1− a2 )...(1− an ) n
                                            IMO Shortlist, 1998
153. Cho hai s th c a, b , a ≠ 0 . Ch ng minh r ng
                                                       1 b
                                         a 2 + b2 +      + ≥ 3.
                                                       a2 a
                                               Austria, 2000
154. Cho a1 , a2 , ..., an > 0 . Ch ng minh r ng

                          a12 a2
                               2
                                      a2    a2
                             + + ... + n−1 + n ≥ a1 + a2 + ... + an .
                          a2 a3        an   a1
                                                   China, 1984
155. Cho x, y, z là các s th c dương th a mãn ñi u ki n xyz = 1 . Ch ng minh r ng
                          x 2 + y 2 + z 2 + x + y + z ≥ 2 ( xy + yz + zx) .
                                                Russia, 2000
156. Cho x, y, z là các s th c dương th a mãn ñi u ki n xyz ≥ xy + yz + zx . Ch ng minh
r ng
                                             xyz ≥ 3( x + y + z ) .
                                                   India, 2001
                          1 1 1
157. Cho x, y, z > 1 và    + + = 2 . Ch ng minh r ng
                          x y z
                                 x + y + z ≥ x −1 + y −1 + z −1 .
                                                   IMO, 1992
158. Cho a, b, c là các s th c dương th a mãn ñi u ki n ab +bc +ca =1. Ch ng minh r ng

                                 1         1        1        1
                             3     + 6b + 3 + 6c + 3 + 6a ≤     .
                                 a         b        c       abc
18
500 Bài Toán B t ð ng Th c Ch n L c                                           Cao Minh Quang

                                      IMO Shortlist, 2004
159. Cho x ≥ 2, y ≥ 2, z ≥ 2 . Ch ng minh r ng

                             ( x3 + y )( y 3 + z )( z 3 + x) ≥ 125 xyz .
                                   Saint Petersburg, 1997

160. Cho a, b, c, d là các s th c dương th a mãn ñi u ki n c 2 + d 2 = (a 2 + b 2 ) . Ch ng
                                                                                    3


minh r ng
                                            a 3 b3
                                               + ≥ 1.
                                            c   d
                                         Singapore, 2000
161. Cho a, b, c là các s th c dương. Ch ng minh r ng
                                  a       b      c
                                      +      +       ≥1.
                                b + 2c c + 2a a + 2b
                                Czech – Slovak Match, 1999
162. Cho a, b, c là các s th c dương. Ch ng minh r ng
                       ab       bc        ca       a     b     c
                            +         +         ≥     +     +      .
                   c (c + a) a (a + b) b (b + c) c + a b + a c + b
                                         Moldova, 1999
163. Cho a, b, c, d là các s th c dương. Ch ng minh r ng
                             a +c b+d c +a d +b
                                 +   +    +     ≥ 4.
                             a+b b+c c +d d +a
                                        Baltic way, 1995
164. Cho x, y, u , v là các s th c dương. Ch ng minh r ng
                             xy + xu + uy + uv    xy   uv
                                               ≥     +     .
                               x + y +u +v       x+ y u +v
                                           Poland, 1993
165. Cho a, b, c là các s th c dương. Ch ng minh r ng
                           a  b  c                       
                          1 + 1 + 1 +  ≥ 2 1 + a + b + c  .
                                                                
                           b  c  a 
                          
                              
                                   
                                         
                                           
                                                 
                                                 
                                                       3
                                                          abc  

                                          APMO, 1998
166. Cho x, y, z là các s th c không âm th a mãn ñi u ki n x + y + z =1. Ch ng minh r ng
                                                                4
                                      x2 y + y 2 z + z 2 x ≤      .
                                                               27
                                          Canada, 1999
167. Cho a, b, c, d , e, f là các s th c dương th a mãn ñi u ki n
                                                                       1
                         a + b + c + d + e + f = 1, ace + bdf ≥           .
                                                                      108
     Ch ng minh r ng


                                                                                           19
500 Bài Toán B t ð ng Th c Ch n L c                                                                          Cao Minh Quang

                                                                                                       1
                             abc + bcd + cde + def + efa + fab ≤                                         .
                                                                                                      36
                                                            Poland, 1998
168. Cho a, b, c ∈ [0,1] . Ch ng minh r ng

                             a 2 + b 2 + c 2 ≤ a 2b + b 2 c + c 2 a + 1 .
                                                      Italy, 1993
169. Cho a, b, c ≥ 0, a + b + c ≥ abc . Ch ng minh r ng
                                             a 2 + b 2 + c 2 ≥ abc .
                                                    Ireland, 1997
170. Cho a, b, c ≥ 0, a + b + c ≥ abc . Ch ng minh r ng

                                            a 2 + b 2 + c 2 ≥ 3abc .
                                                     BMO, 2001
171. Cho x, y, z là các s th c dương th a mãn ñi u ki n x + y + z = xyz . Ch ng minh r ng

                                    xy + yz + zx ≥ 9 ( x + y + z ) .
                                                   Belarus, 1996
172. Cho x1 , x2 , x3 , x4 là các s th c dương th a mãn ñi u ki n x1 x2 x3 x4 = 1 . Ch ng minh
r ng
                                        
                                                            1  1  1  1 
               x13 + x2 + x3 + x4 ≥ max  x1 + x2 + x3 + x4 , + + +  .
                      3    3    3
                                                                        
                                        
                                        
                                                            x1 x2 x3 x4 
                                                                         
                                                                         
                                                      Iran, 1997
173. Cho a, b, c, x, y, z là các s th c dương. Ch ng minh r ng
                                                                                       3
                                    a 3 b3 c 3 (a + b + c )
                                       + + ≥                 .
                                    x    y  z 3( x + y + z )
                                             Belarus TST, 2000
174. Cho a, b, c, d là các s th c dương th a mãn ñi u ki n
                              1        1        1        1
                                 4
                                   +      4
                                            +      4
                                                     +        =1.
                            1+ a     1+ b     1+ c     1+ d 4
     Ch ng minh r ng
                                                        abcd ≥ 3 .
                                                    Latvia, 2002
175. Cho x, y, z > 1 . Ch ng minh r ng
                                  2 +2 yz        2 + 2 zx        2 +2 xy           xy + yz + zx
                             xx             yy              zz             ≥ ( xyz )              .
                                   Proposed for 1999 USAMO
176. Cho c ≥ b ≥ a ≥ 0 . Ch ng minh r ng
                             (a + 3b)(b + 4c)(c + 2a) ≥ 60abc .
                                                    Turkey, 1999

20
500 Bài Toán B t ð ng Th c Ch n L c                                              Cao Minh Quang

177. Cho x, y, z là các s th c dương. Ch ng minh r ng

                                    x 2 + y 2 + z 2 ≥ 2 ( xy + yz ) .
                                           Macedonia, 2000
178. Cho các s th c a, b, c th a mãn ñi u ki n a 2 + b 2 + c 2 = 1 . Ch ng minh r ng
                                    a2      b2      c2   3
                                        +       +       ≥ .
                                 1 + 2bc 1 + 2ca 1 + 2ab 5
                                  Bosnia and Hercegovina, 2002
179. Cho a, b, c là các s th c dương th a mãn ñi u ki n abc ≥ 1 . Ch ng minh r ng
                               1         1          1
                               4   4
                                     + 4     4
                                               + 4         ≤1.
                            a+b +c    a +b+c    a + b4 + c
                                                Korea, 1999
180. Cho a > b > c > 0, x > y > z > 0 . Ch ng minh r ng
                      a 2 x2               b2 y 2              c2 z 2       3
                                   +                   +                   ≥ .
               (by + cz )(bz + cy ) (cz + ax)(cx + az ) (ax + by )(ay + bx) 4
                                                Korea, 2000
181. Cho a, b, c là các s th c không âm th a mãn ñi u ki n a + b + c = 3 . Ch ng minh
r ng
                                      a      b      c    3
                                         +      +       ≥ .
                                    b2 +1 c 2 +1 a 2 + 1 2
                                       Mediterranean, 2003
182. Cho a, b, c là các s th c dương. Ch ng minh r ng
                                     a      b       c
                                         +      +       ≤1.
                                   2a + b 2b + c 2c + a
                                            Moldova, 2002
183. Cho α, β , x1 , x2 ,..., xn > 0, x1 + x2 + ... + xn = 1 . Ch ng minh r ng
                                                            3
                           x13          3
                                       x2                  xn         1
                                 +            + ... +            ≥           .
                      α x1 + β x2 α x2 + β x3         α xn + β x1 n (α + β )
                                          Moldova TST, 2002
184. Cho a là m t s th c dương, x1 , x2 ,..., xn > 0, x1 + x2 + ... + xn = 1 . Ch ng minh r ng

                               a x1−x2   a x2 −x3         a xn −x1  n2
                                       +          + ... +          ≥ .
                               x1 + x2 x2 + x3            xn + x1   2
                                                Serbia, 1998
185. Cho x, y ∈ [ 0,1] . Ch ng minh r ng
                                      1              1               2
                                                +              ≤          .
                                     1+ x   2
                                                    1+ y   2
                                                                   1 + xy
                                                Russia, 2000



                                                                                                 21
500 Bài Toán B t ð ng Th c Ch n L c                                                     Cao Minh Quang

                               1 1 1
186. Cho x, y , z > 0, xyz = 1, + + > x + y + z, k ∈ N * . Ch ng minh r ng
                               x y z
                                   1     1   1
                                     k
                                       + k + k > xk + y k + z k .
                                   x    y   z
                                            Russia, 1999
187. Cho xn ≥ xn−1 ≥ xn−2 ≥ ... ≥ x1 > 0, n ≥ 3 . Ch ng minh r ng
                           xn x1 x1 x2        x x
                                +      + ... + n−1 n ≥ x1 + x2 + ... + xn .
                            x2    x3            x1
                                      Saint Petersburg, 2000
188. Cho x1 ,..., x6 ∈ [ 0,1] . Ch ng minh r ng
                                                                          3
                   x13                    x23
                                                                        x6            3
           5    5          5
                                + 5    5          5
                                                        + ... + 5    5          5
                                                                                     ≤ .
          x2 + x3 + ... + x6 + 5 x3 + x4 + ... + x1 + 5        x1 + x2 + ... + x5 + 5 5
                                           Ukraine, 1999
189. Cho a1 , a2 ,..., an > 0 . Ch ng minh r ng

                  (a13 +1)(a23 +1)...(an3 +1) ≥ (a12 a2 +1)(a22 a3 +1)...(an2a1 +1) .
                             Czech – Slovak – Polish Match 2001
190. Cho a, b, c là các s th c dương th a mãn ñi u ki n a + b + c = 1 . Ch ng minh r ng

                          a. 3 1 + b − c + b. 3 1 + c − a + c. 3 1 + a − b ≤ 1 .
                                             Japan, 2005
191. Cho a, b, c là các s th c dương. Ch ng minh r ng

                              a b c 2
                                                             
                              + +  ≥ (a + b + c ) 1 + 1 + 1  .
                                                             
                             b c a
                                                 a b c
                                                              
                                              Iran, 2005
192. Cho a, b, c, d là các s th c dương. Ch ng minh r ng
                               1     1  1  1  a+b+c +d
                                 3
                                   + 3+ 3+ 3≥          .
                               a    b  c  d     abcd
                                            Austria, 2005
193. Cho a, b, c ∈ [0,1] . Ch ng minh r ng
                                     a      b      c
                                         +      +       ≤2.
                                   bc + 1 ca + 1 ab + 1
                                            Poland, 2005
194. Cho a, b, c là các s th c dương th a mãn ñi u ki n a + b + c = 1 . Ch ng minh r ng
                                                                1
                                     a b +b c +c a ≤               .
                                                                 3
                                 Bosnia and Hercegovina, 2005
195. Cho a, b, c là các s th c dương th a mãn ñi u ki n a + b + c = 1 . Ch ng minh r ng



22
500 Bài Toán B t ð ng Th c Ch n L c                                                           Cao Minh Quang

                                  b c a  1+ a 1+ b 1+ c
                                2 + +  ≥
                                        
                                               +    +     .
                                 
                                  a b c  1 − a 1− b 1− c
                                               Germany, 2005
196. Cho a, b, c là các s th c dương. Ch ng minh r ng
                                                                                     2
                                a2 b2 c 2        4 ( a − b)
                                  + + ≥ a +b+c +            .
                                b  c  a          a +b+c
                                                   Balkan, 2005
197. Cho a, b, c là các s th c dương th a mãn ñi u ki n abc = 8 . Ch ng minh r ng
                           a2                            b2                         c2   4
                                        +                               +               ≥ .
                 (a   3
                          + 1)(b +1)
                                3
                                              (b   3
                                                       + 1)(c + 1)
                                                                 3
                                                                            (c +1)(a +1) 3
                                                                                3        3



                                                   APMO, 2005
198. Cho a, b, c là các s th c dương th a mãn ñi u ki n abc = 1 . Ch ng minh r ng
                                         a     b     c
                                        2
                                           + 2   + 2   ≤1 .
                                       a +2 b +2 c +2
                                              Baltic way, 2005
199. Cho x, y, z là các s th c dương th a mãn ñi u ki n xyz ≥ 1 . Ch ng minh r ng
                              x5 − x 2       y5 − y 2      z5 − z 2
                                         + 5           + 5            ≥0.
                           x5 + y 2 + z 2 y + z 2 + x 2 z + x 2 + y 3
                                                       IMO, 2005
200. Cho a, b, c là các s th c dương. Ch ng minh r ng
                             2                                     
                            a + b + 3 b 2 + a + 3  ≥ 2a + 1 2b + 1 
                                                                   
                            
                                      
                                     4           4         
                                                               2      2

                                              Belarusian, 2005
                                                                                1 1 1
201. Cho a, b, c là các s th c dương th a mãn ñi u ki n                          + + = 1 . Ch ng minh r ng
                                                                                a b c
                                            (a −1)(b −1)(c −1) ≥ 8
                                                Croatia, 2005
202. Cho x là s th c dương. Ch ng minh r ng
                                                                  (2 x)
                                                                        n
                                                       n +1
                                             1+ x             ≥         n−1
                                                                            .
                                                                (1 + x)
                                                   Russia, 2005
203. Cho a, b, c là các s th c dương th a mãn ñi u ki n abc ≥ 1 . Ch ng minh r ng
                                       1        1        1
                                            +        +         ≤ 1.
                                    1+ a + b 1+ b + c 1+ c + a
                                               Romania, 2005
204. Cho a, b, c là các s th c dương th a mãn ñi u ki n abc = 1 . Ch ng minh r ng



                                                                                                             23
500 Bài Toán B t ð ng Th c Ch n L c                                                Cao Minh Quang

                               a                   a                 a        3
                                           +                +                ≥ .
                       (a +1)(b +1) (b +1)(c +1)                (c +1)(a + 1) 4
                                       Czech and Slovak, 2005
                                                                         1
205. Cho a, b, c là các s th c không âm th a mãn ñi u ki n ab + bc + ca = . Ch ng minh
                                                                         3
r ng
                             1          1           1
                           2
                                    + 2        + 2         ≤3.
                          a − bc + 1 b − ca + 1 c − ab + 1
                                                China, 2005
206. Cho a, b, c là các s th c dương th a mãn ñi u ki n a + b + c = 1 . Ch ng minh r ng

                                                                           2
                           ab (1− c ) + bc (1− a ) + ca (1− b) ≤             .
                                                                           3
                                      Republic of Srpska, 2005
207. Cho a, b, c là các s th c dương. Ch ng minh r ng

                            a     b     c     3
                               +     +      ≥   (a + b + c) .
                           b+c   c+a   a +b   2
                                   Serbia and Montenegro, 2005
208. Cho a, b, c là các s th c dương th a mãn ñi u ki n a 4 + b 4 + c 4 = 3 . Ch ng minh
r ng
                                        1     1      1
                                           +      +       ≤1 .
                                     4 − ab 4 − bc 4 − ca
                                               Moldova, 2005
209. Cho a, b, c là các s th c dương th a mãn ñi u ki n ab +bc +ca =1. Ch ng minh r ng
                                                                 3
                                            1                      3
                                    3. 3       + 6 (a + b + c) ≤     .
                                           abc                   abc
                                            Slovenia TST, 2005
210. Cho a, b, c là các s th c dương th a mãn ñi u ki n a, b, c ≥ 1 . Ch ng minh r ng
                                                  1    1   1
                                      (2 + abc) + +  ≥ 9 .
                                               a b c
                                                    
                                                    
211. [ Huỳnh T n Châu ] Cho x, y, z là các s th c dương th a mãn ñi u ki n

                                    xy xy + yz yz + zx zx = 1 .
     Ch ng minh r ng
                                      x6       y6       z6    1
                                    3    3
                                           + 3    3
                                                    + 3    3
                                                             ≥ .
                                   x +y     y +z     z +x     2
212. [ ð ng Thanh H i ] Cho x là m t s th c b t kì. Ch ng minh r ng
                                                                  3 3
                                    sin x + sin 2 x + sin 3 x <       .
                                                                   2
213. [ Ngô Văn Thái ] Cho x1 , x2 ,..., xn > 0, n > 2 . Ch ng minh r ng

24
500 Bài Toán B t ð ng Th c Ch n L c                                                     Cao Minh Quang

                x12 + x2 x3      2
                               x2 + x3 x4              2
                                                      xn−1 + xn x1       2
                                                                       xn + x1 x2
                             +               + ... +                 +               ≥n.
               x1 ( x2 + x3 ) x2 ( x3 + x4 )         xn−1 ( xn + x1 ) xn ( x1 + x2 )

214. [ Nguy n Duy Liên ] Cho a, b, c là các s th c dương th a mãn ñi u ki n a, b, c ∈ [1, 2] .
Ch ng minh r ng
                                                   1      1      1
                                   (a + b + c) + +  ≤ 10 .
                                              
                                                   
                                                    
                                                   a      b      c
215. [ Lê Thanh H i ] Cho a, b, c d là các s th c dương. Ch ng minh r ng
                                a2 b2 c 2 d 2 a + b + c + d
                                  + + +      ≥              .
                                b2 c2 d 2 a2     4
                                                   abcd
216. Cho x ∈ [0, 2] . Ch ng minh r ng

                                       4 x − x3 + x + x3 ≤ 3 4 3 .
217. Cho x là m t s th c b t kì. Ch ng minh r ng

                                   2 sin x + 15 −10 2 cos x ≤ 6 .
218. [ Tr n Văn H nh ] Cho x, y, z là các s th c dương th a mãn ñi u ki n x2 + y2 + z2 =1,
n ≥ 1 . Ch ng minh r ng

                           x         y        z        (2n + 1) 2 n 2n +1
                                 +         +         ≥                    .
                        1 − x 2 n 1 − y 2 n 1− z 2 n          2n
219. [ Ki u Phương Chi ] Cho a, b, c là các s th c dương th a mãn ñi u ki n abc = 1 .
Ch ng minh r ng
                           1          1          1      1
                         2   2
                                 + 2    2
                                            + 2    2
                                                       ≤ .
                       a + 2b + 3 b + 2c + 3 c + 2a + 3 2
220. [ Vũ ð c C nh ] Cho x, y là các s th c dương th a mãn ñi u ki n x 2 + y 2 = 1 . Ch ng
minh r ng
                                          1
                                                       1
                                    
                             (1 + x)1 +                    
                                            + (1 + y )1 +  ≥ 4 + 3 2 .
                                           
                                                      
                                                       
                                                           
                                          y              x
221. [ Ngô Văn Thái ] Cho a, b, c ∈ (0,1] . Ch ng minh r ng
                                   1    1
                                       ≥ + (1− a )(1− b)(1− c) .
                                a +b +c 3
222. [ Nguy n Văn Thông ] Cho x, y, z là các s th c dương th a mãn ñi u ki n
                                        3x   4y   2z
                                           +    +      = 2.
                                       x +1 y +1 z + 1
   Ch ng minh r ng
                                                               1
                                                x3 y 4 z 2 ≤      .
                                                               89
223. [ Nguy n Bá Nam ] Cho a, b, c là các s th c dương. Ch ng minh r ng
                                   1       1       1         3 b + c c + a a + b
                  (a3 + b3 + c3 ) a3 + b3 + c3  ≥ 2 
                                 
                                 
                                 
                                                
                                                
                                                     
                                                      
                                                                   a
                                                                       +
                                                                          b
                                                                             +
                                                                                c 
                                                                                   .
                                                                                   
                                                                                   


                                                                                                     25
500 Bài Toán B t ð ng Th c Ch n L c                                                 Cao Minh Quang

224. Cho x là m t s th c b t kì. Ch ng minh r ng

                              (16 cos4 x + 3)
                                              4
                                                  + 768 ≥ 2048cos x .

225. [ Lê Qu c Hán ] Cho x là m t s th c b t kì. Ch ng minh r ng
                                                  8
                                    1 (1 + x ) +16 x
                                                     4
                                      ≤                ≤ 17 .
                                         (1+ x2 )
                                                 4
                                    8

226. [ Nguy n Lê Dũng ] Cho a, b, c là các s th c dương. Ch ng minh r ng
                        a2 + b2 b2 + c2 c2 + a2    a2 + b2 + c2
                               +       +        ≤3              .
                         a +b    b+c     c+a        a +b+c
227. [ Tr n Xuân ðáng ] Cho a, b, c là các s th c dương, n ≥ 2 . Ch ng minh r ng

                              a      b       c     n n
                         n       +n     +n      >      n −1 .
                             b+c    c+a    a + b n −1
228. [ Tr nh B ng Giang ] Cho x, y, z là các s th c không âm th a ñi u ki n x + y + z = 1 ,
n ≥ 2 . Ch ng minh r ng
                                                               nn
                                 xn y + y n z + z n x ≤             n +1
                                                                           .
                                                          (n + 1)
229. [ Nguy n Văn Ng c ] Cho x, y, z là các s th c dương. Ch ng minh r ng
                                                           4               4   4
                      16 xyz ( x + y + z ) ≤ 3 3 ( x + y ) ( y + z ) ( z + x ) .

                                       π π
230. [ Nguy n Bá ðang ] Cho x, y , z ∈  ,  . Ch ng minh r ng
                                        6 2 

                    sin x − sin y sin y − sin z sin z − sin x       2
                                                                   1 
                                 +             +             ≤ 1−
                                                                     .
                        sin z         sin x         sin y      
                                                                   2
                                                                     
231. [ Thái Nh t Phư ng ] Cho x, y, z là các s th c dương th a mãn ñi u ki n xyz = 1 .
Ch ng minh r ng
                            x2            y2             z2
                                     +             +             ≥ 3.
                        x + y + y 3 z y + z + z 3 x z + x + x3 y
232. [ Thái Nh t Phư ng ] Cho x, y, z là các s th c dương th a mãn ñi u ki n xyz = 1 .
Ch ng minh r ng
                           x2 y 2          y2 z2           z 2 x2
                                     + 2 2           + 2 2           ≤1 .
                   x 2 y 2 + x7 + y 7 y z + y 7 + z 7 z x + z 7 + x7
233. [ Trương Ng c ð c ] Cho a, b, c là các s th c dương th a mãn ñi u ki n a +b +c =1.
Ch ng minh r ng
                                a      b      abc       3 3
                                    +      +       ≤ 1+     .
                              a + bc b + ca c + ab       4
234. [ Nguy n Minh Phương ] Cho x, y, z là các s                      th c dương th a mãn ñi u ki n
x + y + z = 2007 . Ch ng minh r ng
                                   x 20 y 20 z 20
                                     11
                                        + 11 + 11 ≥ 3.6699 .
                                   y     z    x
26
500 bdt
500 bdt
500 bdt
500 bdt
500 bdt
500 bdt
500 bdt
500 bdt
500 bdt
500 bdt
500 bdt
500 bdt
500 bdt
500 bdt
500 bdt
500 bdt
500 bdt
500 bdt
500 bdt
500 bdt
500 bdt
500 bdt
500 bdt

More Related Content

What's hot

Bdt dt chuyen_qt_l_t
Bdt dt chuyen_qt_l_tBdt dt chuyen_qt_l_t
Bdt dt chuyen_qt_l_t
Tam Vu Minh
 
[Www.toan trunghoccoso.toancapba.net] toan 9 chuyen de bd hsg nang khieu
[Www.toan trunghoccoso.toancapba.net] toan 9  chuyen de bd hsg  nang khieu[Www.toan trunghoccoso.toancapba.net] toan 9  chuyen de bd hsg  nang khieu
[Www.toan trunghoccoso.toancapba.net] toan 9 chuyen de bd hsg nang khieu
Tam Vu Minh
 
Thi thử toán trần nguyên hãn hp 2012 lần 1
Thi thử toán trần nguyên hãn hp 2012 lần 1Thi thử toán trần nguyên hãn hp 2012 lần 1
Thi thử toán trần nguyên hãn hp 2012 lần 1
Thế Giới Tinh Hoa
 
6856978 collected-problems-about-inequality
6856978 collected-problems-about-inequality6856978 collected-problems-about-inequality
6856978 collected-problems-about-inequality
ria_nghia
 
9dethithu
9dethithu9dethithu
9dethithu
Duy Duy
 
Tuyen tap cac_bat_dang_thuc_trong_cac_de_thi_tuyen_sing_dai_hoc(ca_hd)
Tuyen tap cac_bat_dang_thuc_trong_cac_de_thi_tuyen_sing_dai_hoc(ca_hd)Tuyen tap cac_bat_dang_thuc_trong_cac_de_thi_tuyen_sing_dai_hoc(ca_hd)
Tuyen tap cac_bat_dang_thuc_trong_cac_de_thi_tuyen_sing_dai_hoc(ca_hd)
Nguyen KienHuyen
 
Thi thử toán chuyên nguyễn quang diêu đt 2012 lần 2 k d
Thi thử toán chuyên nguyễn quang diêu đt 2012 lần 2 k dThi thử toán chuyên nguyễn quang diêu đt 2012 lần 2 k d
Thi thử toán chuyên nguyễn quang diêu đt 2012 lần 2 k d
Thế Giới Tinh Hoa
 
221 bat dang thuc
221 bat dang thuc221 bat dang thuc
221 bat dang thuc
ongdongheo
 
Deonmontoanso4
Deonmontoanso4Deonmontoanso4
Deonmontoanso4
Duy Duy
 
[Vnmath.com] dethi-dapan-thivao-10-toan-2012-2013-cac-tinh
[Vnmath.com] dethi-dapan-thivao-10-toan-2012-2013-cac-tinh[Vnmath.com] dethi-dapan-thivao-10-toan-2012-2013-cac-tinh
[Vnmath.com] dethi-dapan-thivao-10-toan-2012-2013-cac-tinh
Khoa Tuấn
 

What's hot (17)

Bdt dt chuyen_qt_l_t
Bdt dt chuyen_qt_l_tBdt dt chuyen_qt_l_t
Bdt dt chuyen_qt_l_t
 
[Www.toan trunghoccoso.toancapba.net] toan 9 chuyen de bd hsg nang khieu
[Www.toan trunghoccoso.toancapba.net] toan 9  chuyen de bd hsg  nang khieu[Www.toan trunghoccoso.toancapba.net] toan 9  chuyen de bd hsg  nang khieu
[Www.toan trunghoccoso.toancapba.net] toan 9 chuyen de bd hsg nang khieu
 
Thi thử toán trần nguyên hãn hp 2012 lần 1
Thi thử toán trần nguyên hãn hp 2012 lần 1Thi thử toán trần nguyên hãn hp 2012 lần 1
Thi thử toán trần nguyên hãn hp 2012 lần 1
 
6856978 collected-problems-about-inequality
6856978 collected-problems-about-inequality6856978 collected-problems-about-inequality
6856978 collected-problems-about-inequality
 
9dethithu
9dethithu9dethithu
9dethithu
 
Chuyen de bat_dang_thuc
Chuyen de bat_dang_thucChuyen de bat_dang_thuc
Chuyen de bat_dang_thuc
 
Bdt võ quốc bá cẩn
Bdt  võ quốc bá cẩnBdt  võ quốc bá cẩn
Bdt võ quốc bá cẩn
 
Tuyen tap cac_bat_dang_thuc_trong_cac_de_thi_tuyen_sing_dai_hoc(ca_hd)
Tuyen tap cac_bat_dang_thuc_trong_cac_de_thi_tuyen_sing_dai_hoc(ca_hd)Tuyen tap cac_bat_dang_thuc_trong_cac_de_thi_tuyen_sing_dai_hoc(ca_hd)
Tuyen tap cac_bat_dang_thuc_trong_cac_de_thi_tuyen_sing_dai_hoc(ca_hd)
 
Thi thử toán chuyên nguyễn quang diêu đt 2012 lần 2 k d
Thi thử toán chuyên nguyễn quang diêu đt 2012 lần 2 k dThi thử toán chuyên nguyễn quang diêu đt 2012 lần 2 k d
Thi thử toán chuyên nguyễn quang diêu đt 2012 lần 2 k d
 
Tuyển tập 300 bài bdt
Tuyển tập 300 bài bdtTuyển tập 300 bài bdt
Tuyển tập 300 bài bdt
 
Chuyen de hoc sinh giỏi toan9
Chuyen de hoc sinh giỏi toan9Chuyen de hoc sinh giỏi toan9
Chuyen de hoc sinh giỏi toan9
 
Bdt thuần nhất
Bdt thuần nhấtBdt thuần nhất
Bdt thuần nhất
 
Bat dang thuc don bien co dien
Bat dang thuc don bien co dienBat dang thuc don bien co dien
Bat dang thuc don bien co dien
 
221 bat dang thuc
221 bat dang thuc221 bat dang thuc
221 bat dang thuc
 
Bdt bunhiacopski
Bdt bunhiacopskiBdt bunhiacopski
Bdt bunhiacopski
 
Deonmontoanso4
Deonmontoanso4Deonmontoanso4
Deonmontoanso4
 
[Vnmath.com] dethi-dapan-thivao-10-toan-2012-2013-cac-tinh
[Vnmath.com] dethi-dapan-thivao-10-toan-2012-2013-cac-tinh[Vnmath.com] dethi-dapan-thivao-10-toan-2012-2013-cac-tinh
[Vnmath.com] dethi-dapan-thivao-10-toan-2012-2013-cac-tinh
 

Similar to 500 bdt

15 bai toan_boi_duong_hsg_toan_l8_8383
15 bai toan_boi_duong_hsg_toan_l8_838315 bai toan_boi_duong_hsg_toan_l8_8383
15 bai toan_boi_duong_hsg_toan_l8_8383
Manh Tranduongquoc
 
Thi thử toán thuận thành 1 bn 2012 lần 2
Thi thử toán thuận thành 1 bn 2012 lần 2Thi thử toán thuận thành 1 bn 2012 lần 2
Thi thử toán thuận thành 1 bn 2012 lần 2
Thế Giới Tinh Hoa
 
19 phương phap chứng minh bất đẳng thức
19 phương phap chứng minh bất đẳng thức19 phương phap chứng minh bất đẳng thức
19 phương phap chứng minh bất đẳng thức
Thế Giới Tinh Hoa
 
[Hsgs.edu.vn] tuyen tap 300 bai bat dang thuc suu tam tu cac dien dan toan hoc
[Hsgs.edu.vn] tuyen tap 300 bai bat dang thuc suu tam tu cac dien dan toan hoc[Hsgs.edu.vn] tuyen tap 300 bai bat dang thuc suu tam tu cac dien dan toan hoc
[Hsgs.edu.vn] tuyen tap 300 bai bat dang thuc suu tam tu cac dien dan toan hoc
Tam Vu Minh
 
Tuyntp300bibdt 121016083537-phpapp02 (1)
Tuyntp300bibdt 121016083537-phpapp02 (1)Tuyntp300bibdt 121016083537-phpapp02 (1)
Tuyntp300bibdt 121016083537-phpapp02 (1)
Marco Reus Le
 
19 phương pháp chứng minh bđt
19 phương pháp chứng minh bđt19 phương pháp chứng minh bđt
19 phương pháp chứng minh bđt
Cảnh
 
Vecto.laisac
Vecto.laisacVecto.laisac
Vecto.laisac
laisac
 

Similar to 500 bdt (20)

Bộ sưu tập bất đẳng thức của võ quốc bá cẩn
Bộ sưu tập bất đẳng thức của võ quốc bá cẩnBộ sưu tập bất đẳng thức của võ quốc bá cẩn
Bộ sưu tập bất đẳng thức của võ quốc bá cẩn
 
500 bdt thay cao minh quang tong hop
500 bdt thay cao minh quang tong hop500 bdt thay cao minh quang tong hop
500 bdt thay cao minh quang tong hop
 
550 bdt-chon-loc (1)
550 bdt-chon-loc (1)550 bdt-chon-loc (1)
550 bdt-chon-loc (1)
 
500 bat dang thuc tong hop cao minh quang
500 bat dang thuc tong hop   cao minh quang500 bat dang thuc tong hop   cao minh quang
500 bat dang thuc tong hop cao minh quang
 
15 bai toan_boi_duong_hsg_toan_l8_8383
15 bai toan_boi_duong_hsg_toan_l8_838315 bai toan_boi_duong_hsg_toan_l8_8383
15 bai toan_boi_duong_hsg_toan_l8_8383
 
19 phuong phap chung minh bat dang thu ccb
 19 phuong phap chung minh bat dang thu ccb 19 phuong phap chung minh bat dang thu ccb
19 phuong phap chung minh bat dang thu ccb
 
Ds10 c4a
Ds10 c4aDs10 c4a
Ds10 c4a
 
Cm bat dang thuc bang pp tiep tuyen
Cm bat dang thuc bang pp tiep tuyenCm bat dang thuc bang pp tiep tuyen
Cm bat dang thuc bang pp tiep tuyen
 
Thi thử toán thuận thành 1 bn 2012 lần 2
Thi thử toán thuận thành 1 bn 2012 lần 2Thi thử toán thuận thành 1 bn 2012 lần 2
Thi thử toán thuận thành 1 bn 2012 lần 2
 
Chuyen de bat dang thuc cauchy
Chuyen de bat dang thuc cauchyChuyen de bat dang thuc cauchy
Chuyen de bat dang thuc cauchy
 
Bai tap-nang-cao-toan-hinh-lop-8-bai-tap-nang-cao-toan-dai-lop-8
Bai tap-nang-cao-toan-hinh-lop-8-bai-tap-nang-cao-toan-dai-lop-8Bai tap-nang-cao-toan-hinh-lop-8-bai-tap-nang-cao-toan-dai-lop-8
Bai tap-nang-cao-toan-hinh-lop-8-bai-tap-nang-cao-toan-dai-lop-8
 
Bai tap-nang-cao-toan-hinh-lop-8-bai-tap-nang-cao-toan-dai-lop-8
Bai tap-nang-cao-toan-hinh-lop-8-bai-tap-nang-cao-toan-dai-lop-8Bai tap-nang-cao-toan-hinh-lop-8-bai-tap-nang-cao-toan-dai-lop-8
Bai tap-nang-cao-toan-hinh-lop-8-bai-tap-nang-cao-toan-dai-lop-8
 
19 phương phap chứng minh bất đẳng thức
19 phương phap chứng minh bất đẳng thức19 phương phap chứng minh bất đẳng thức
19 phương phap chứng minh bất đẳng thức
 
Batdangthuc nesbitt
Batdangthuc nesbittBatdangthuc nesbitt
Batdangthuc nesbitt
 
đề Thi cấp huyện
đề Thi cấp huyệnđề Thi cấp huyện
đề Thi cấp huyện
 
[Hsgs.edu.vn] tuyen tap 300 bai bat dang thuc suu tam tu cac dien dan toan hoc
[Hsgs.edu.vn] tuyen tap 300 bai bat dang thuc suu tam tu cac dien dan toan hoc[Hsgs.edu.vn] tuyen tap 300 bai bat dang thuc suu tam tu cac dien dan toan hoc
[Hsgs.edu.vn] tuyen tap 300 bai bat dang thuc suu tam tu cac dien dan toan hoc
 
Tuyntp300bibdt 121016083537-phpapp02 (1)
Tuyntp300bibdt 121016083537-phpapp02 (1)Tuyntp300bibdt 121016083537-phpapp02 (1)
Tuyntp300bibdt 121016083537-phpapp02 (1)
 
Chuyen de can thuc bac hai
Chuyen de can thuc bac haiChuyen de can thuc bac hai
Chuyen de can thuc bac hai
 
19 phương pháp chứng minh bđt
19 phương pháp chứng minh bđt19 phương pháp chứng minh bđt
19 phương pháp chứng minh bđt
 
Vecto.laisac
Vecto.laisacVecto.laisac
Vecto.laisac
 

More from Thế Giới Tinh Hoa

Cách tắm cho bé vào mùa đông
Cách tắm cho bé vào mùa đôngCách tắm cho bé vào mùa đông
Cách tắm cho bé vào mùa đông
Thế Giới Tinh Hoa
 
Nữ quái sân trườngtruonghocso.com
Nữ quái sân trườngtruonghocso.comNữ quái sân trườngtruonghocso.com
Nữ quái sân trườngtruonghocso.com
Thế Giới Tinh Hoa
 
Những chàng trai xấu tính nguyễn nhật ánhtruonghocso.com
Những chàng trai xấu tính  nguyễn nhật ánhtruonghocso.comNhững chàng trai xấu tính  nguyễn nhật ánhtruonghocso.com
Những chàng trai xấu tính nguyễn nhật ánhtruonghocso.com
Thế Giới Tinh Hoa
 
Những bài văn mẫu dành cho học sinh lớp 10truonghocso.com
Những bài văn mẫu dành cho học sinh lớp 10truonghocso.comNhững bài văn mẫu dành cho học sinh lớp 10truonghocso.com
Những bài văn mẫu dành cho học sinh lớp 10truonghocso.com
Thế Giới Tinh Hoa
 

More from Thế Giới Tinh Hoa (20)

Cách chụp ảnh công ty đẹp 2019
Cách chụp ảnh công ty đẹp 2019Cách chụp ảnh công ty đẹp 2019
Cách chụp ảnh công ty đẹp 2019
 
Lỗi web bachawater
Lỗi web bachawaterLỗi web bachawater
Lỗi web bachawater
 
Bảng báo giá sản phẩm rèm bạch dương
Bảng báo giá sản phẩm rèm bạch dươngBảng báo giá sản phẩm rèm bạch dương
Bảng báo giá sản phẩm rèm bạch dương
 
Album sổ mẫu Rèm cửa Bạch Dương
Album sổ mẫu Rèm cửa Bạch DươngAlbum sổ mẫu Rèm cửa Bạch Dương
Album sổ mẫu Rèm cửa Bạch Dương
 
thong tin lam viec tren lamchame
thong tin lam viec tren lamchamethong tin lam viec tren lamchame
thong tin lam viec tren lamchame
 
Cách tắm cho bé vào mùa đông
Cách tắm cho bé vào mùa đôngCách tắm cho bé vào mùa đông
Cách tắm cho bé vào mùa đông
 
Giáo trình tự học illustrator cs6
Giáo trình tự học illustrator cs6  Giáo trình tự học illustrator cs6
Giáo trình tự học illustrator cs6
 
Nang luc truyen thong
Nang luc truyen thongNang luc truyen thong
Nang luc truyen thong
 
Huongdansudung izishop
Huongdansudung izishopHuongdansudung izishop
Huongdansudung izishop
 
Ho so nang luc cong ty
Ho so nang luc cong tyHo so nang luc cong ty
Ho so nang luc cong ty
 
seo contract
seo contractseo contract
seo contract
 
di google cong
di google congdi google cong
di google cong
 
E1 f4 bộ binh
E1 f4 bộ binhE1 f4 bộ binh
E1 f4 bộ binh
 
E2 f2 bộ binh
E2 f2 bộ binhE2 f2 bộ binh
E2 f2 bộ binh
 
E3 f1 bộ binh
E3 f1 bộ binhE3 f1 bộ binh
E3 f1 bộ binh
 
E2 f1 bộ binh
E2 f1 bộ binhE2 f1 bộ binh
E2 f1 bộ binh
 
E1 f1 bộ binh
E1 f1 bộ binhE1 f1 bộ binh
E1 f1 bộ binh
 
Nữ quái sân trườngtruonghocso.com
Nữ quái sân trườngtruonghocso.comNữ quái sân trườngtruonghocso.com
Nữ quái sân trườngtruonghocso.com
 
Những chàng trai xấu tính nguyễn nhật ánhtruonghocso.com
Những chàng trai xấu tính  nguyễn nhật ánhtruonghocso.comNhững chàng trai xấu tính  nguyễn nhật ánhtruonghocso.com
Những chàng trai xấu tính nguyễn nhật ánhtruonghocso.com
 
Những bài văn mẫu dành cho học sinh lớp 10truonghocso.com
Những bài văn mẫu dành cho học sinh lớp 10truonghocso.comNhững bài văn mẫu dành cho học sinh lớp 10truonghocso.com
Những bài văn mẫu dành cho học sinh lớp 10truonghocso.com
 

500 bdt

  • 1. 500 Bài Toán B t ð ng Th c Ch n L c Cao Minh Quang ♦♦♦♦♦ Vĩnh Long, Xuân M u Tý, 2008
  • 2. 500 Bài Toán B t ð ng Th c Ch n L c Cao Minh Quang 500 Bài Toán B t ð ng Th c Ch n L c ♦♦♦♦♦ 1. Cho a, b, c là các s th c dương. Ch ng minh r ng 2 2 2 3 2 a 2 + (1− b) + b 2 + (1− c) + c 2 + (1− a ) ≥ . 2 Komal 2. [ Dinu Serbănescu ] Cho a, b, c ∈ (0,1) . Ch ng minh r ng abc + (1− a )(1− b)(1− c) < 1 . Junior TST 2002, Romania 3. [ Mircea Lascu ] Cho a, b, c là các s th c dương th a mãn ñi u ki n abc = 1 . Ch ng minh r ng b+c c +a a +b + + ≥ a + b + c + 3. a b c Gazeta Matematică 4. N u phương trình x 4 + ax3 + 2 x 2 + bx + 1 = 0 có ít nh t m t nghi m th c, thì a 2 + b2 ≥ 8 . Tournament of the Towns, 1993 5. Cho các s th c x, y, z th a mãn ñi u ki n x 2 + y 2 + z 2 = 1 . Hãy tìm giá tr l n nh t c a bi u th c x3 + y 3 + z 3 − 3xyz . 6. Cho a, b, c, x, y, z là các s th c dương th a mãn ñi u ki n x + y + z = 1 . Ch ng minh r ng ax + by + cz + 2 ( xy + yz + zx )(ab + bc + ca ) ≤ a + b + c . Ukraine, 2001 7. [ Darij Grinberg] Cho a, b, c là các s th c dương. Ch ng minh r ng a b c 9 + + ≥ . (b + c) 2 (c + a ) 2 2 ( a + b) 4 (a + b + c) 8. [ Hojoo Lee ] Cho a, b, c ≥ 0 . Ch ng minh r ng a4 + a2b2 + b4 + b4 + b2c2 + c4 + c4 + c2a2 + a4 ≥ a 2a2 + bc + b 2b2 + ca + c 2c2 + ab . Gazeta Matematică 9. Cho a, b, c là các s th c dương th a mãn ñi u ki n abc = 2 . Ch ng minh r ng a 3 + b 3 + c3 ≥ a b + c + b c + a + c a + b . JBMO 2002 Shortlist 10. [ Ioan Tomescu ] Cho x, y, z là các s th c dương. Ch ng minh r ng xyz 1 ≤ 4. (1 + 3x)( x + 8 y )( y + 9 z )( z + 6) 7 2
  • 3. 500 Bài Toán B t ð ng Th c Ch n L c Cao Minh Quang Gazeta Matematică 11. [ Mihai Piticari, Dan Popescu ] Cho a, b, c là các s th c dương th a mãn ñi u ki n a + b + c = 1 . Ch ng minh r ng 5 (a 2 + b 2 + c 2 ) ≤ 6 (a 3 + b 3 + c3 ) +1 . 12. [ Mircea Lascu ] Cho x1 , x2 ,..., xn ∈ ℝ , n ≥ 2, a > 0 sao cho a2 x1 + x2 + ... + xn = a, x12 + x2 + ... + xn ≤ 2 2 . n −1 Ch ng minh r ng  2a  xi ∈ 0,  , i = 1, 2,..., n .  n  13. [ Adrian Zahariuc ] Cho a, b, c ∈ (0,1) . Ch ng minh r ng b a c b a c + + ≥1 . 4b c − c a 4c a − a b 4a b − b c 14. Cho a, b, c là các s th c dương th a mãn ñi u ki n abc ≤ 1 . Ch ng minh r ng a b c + + ≥ a +b+c . b c a 15. [ Vasile Cirtoaje, Mircea Lascu ] Cho a, b, c, x, y, z là các s th c dương th a mãn ñi u ki n a + x ≥ b + y ≥ c + z , a + b + c = x + y + z . Ch ng minh r ng ay + bx ≥ ac + xz . 16. [ Vasile Cirtoaje, Mircea Lascu ] Cho a, b, c là các s th c dương th a mãn ñi u ki n abc = 1 . Ch ng minh r ng 3 6 1+ ≥ . a + b + c ab + bc + ca Junior TST 2003, Romania 17. Cho a, b, c là các s th c dương. Ch ng minh r ng a 3 b3 c 3 a 2 b 2 c 2 + + ≥ + + . b2 c2 a 2 b c a JBMO 2002 Shortlist 18. Cho x1 , x2 ,..., xn > 0, n > 3 th a mãn ñi u ki n x1 x2 ...xn = 1 . Ch ng minh r ng 1 1 1 + + ... + >1. 1 + x1 + x1 x2 1 + x2 x3 1 + xn + xn x1 Russia, 2004 19. [ Marian Tetiva ] Cho x, y, z là các s th c dương th a ñi u ki n x 2 + y 2 + z 2 + 2 xyz = 1 . Ch ng minh r ng 1 a) xyz ≤ , 8 3 b) x + y + z ≤ , 2 3
  • 4. 500 Bài Toán B t ð ng Th c Ch n L c Cao Minh Quang 3 c) xy + yz + zx ≤ ≤ x 2 + y 2 + z 2 , 4 1 d) xy + yz + zx ≤ + 2 xyz . 2 20. [ Marius Olteanu ] Cho x1 , x2 ,..., x5 ∈ ℝ sao cho x1 + x2 + ... + x5 = 0 . Ch ng minh r ng cos x1 + cos x2 + ... + cos x5 ≥ 1 . Gazeta Matematică 21. [ Florina Cârlan, Marian Tetiva ] Cho x, y, z là các s th c dương th a mãn ñi u ki n x + y + z = xyz . Ch ng minh r ng xy + yz + zx ≥ 3 + x 2 + 1 + y 2 + 1 + z 2 + 1 . 22. [ Laurentiu Panaitopol ] Cho x, y, z là các s th c th a mãn ñi u ki n x, y , z > −1 . Ch ng minh r ng 1+ x2 1+ y2 1+ z 2 + + ≥2. 1+ y + z 2 1+ z + x2 1+ x + y 2 JBMO, 2003 23. Cho a, b, c là các s th c dương th a mãn ñi u ki n a + b + c = 1 . Ch ng minh r ng a2 + b b2 + c c2 + a + + ≥ 2. b+c c+a a +b 24. Cho a, b, c ≥ 0 th a mãn ñi u ki n a 4 + b 4 + c 4 ≤ 2 (a 2b 2 + b 2 c 2 + c 2 a 2 ) . Ch ng minh r ng a 2 + b 2 + c 2 ≤ 2 (ab + bc + ca ) . Kvant, 1988 25. Cho x1 , x2 ,..., xn > 0, n > 2 th a mãn ñi u ki n 1 1 1 1 + + ... + = . x1 +1998 x2 +1998 xn +1998 1998 Ch ng minh r ng n x1 x2 ...xn ≥ 1998 . n −1 Vietnam, 1998 26. [Marian Tetiva ] Cho x, y, z là các s th c dương th a mãn ñi u ki n x 2 + y 2 + z 2 = xyz . Ch ng minh r ng a) xyz ≥ 27, b) xy + yz + zx ≥ 27 , c) x + y + z ≥ 9 , d) xy + yz + zx ≥ 2 ( x + y + z ) + 9 . 27. Cho x, y, z là các s th c dương th a mãn ñi u ki n x + y + z = 3 . Ch ng minh r ng x + y + z ≥ xy + yz + zx . 4
  • 5. 500 Bài Toán B t ð ng Th c Ch n L c Cao Minh Quang Russia 2002 28. [ D. Olteanu ] Cho a, b, c là các s th c dương. Ch ng minh r ng a+b a b+c b c+a c 3 . + . + . ≥ . b + c 2a + b + c c + a 2b + c + a a + b 2c + a + b 4 Gazeta Matematică 29. Cho a, b, c là các s th c dương. Ch ng minh r ng a b c c +a a+b b+c + + ≥ + + . b c a c +b a +c b+a India, 2002 30. Cho a, b, c là các s th c dương. Ch ng minh r ng a3 b3 c3 3(ab + bc + ca) 2 2 + 2 2 + 2 2 ≥ . b − bc + c c − ac + a a − ab + b a +b +c Proposed for the Balkan Mathematical Olympical 31. [ Adrian Zahariuc ] Cho x1 , x2 ,..., xn là các s nguyên ñôi m t phân bi t nhau. Ch ng minh r ng x12 + x2 + ... + xn ≥ x1 x2 + x2 x3 ... + xn x1 + 2n − 3 . 2 2 32. [ Murray Klamkin ] Cho x1 , x2 ,..., xn ≥ 0, n > 2 th a mãn ñi u ki n x1 + x2 + ... + xn = 1 . Hãy tìm giá tr l n nh t c a bi u th c x12 x2 + x2 x3 + ... + xn−1 xn + xn x1 . 2 2 2 Crux Mathematicorum 33. Cho x1 , x2 ,..., xn > 0 th a mãn ñi u ki n xk +1 ≥ x1 + x2 + ... + xk v i m i k. Hãy tìm giá tr l n nh t c a h ng s c sao cho x1 + x2 + ... + xn ≤ c x1 + x2 + ... + xn . IMO Shortlist, 1986 34. Cho các s th c dương a, b, c, x, y, z th a mãn ñi u ki n a + x = b + y = c + z = 1. Ch ng minh r ng 1 1 1   (abc + xyz ) + + ≥ 3.     ay bz cx  Russia, 2002 35. [ Viorel Vâjâitu, Alexvàru Zaharescu ] Cho a, b, c là các s th c dương. Ch ng minh r ng ab bc ca 1 + + ≤ (a + b + c) . a + b + 2c b + c + 2a c + a + 2b 4 Gazeta Matematică 36. Cho a, b, c, d là các s th c th a mãn ñi u ki n a 2 + b 2 + c 2 + d 2 = 1 . Tìm giá tr nh nh t c a bi u th c a 3 (b + c + d ) + b3 (c + d + a) + c 3 (d + a + b) + d 3 (a + b + c) . 37. [ Walther Janous ] Cho x, y, z là các s th c dương. Ch ng minh r ng 5
  • 6. 500 Bài Toán B t ð ng Th c Ch n L c Cao Minh Quang x y z + + ≤1 . x + ( x + y )( x + z ) y + ( y + z )( y + x ) z + ( z + x)( z + y ) Crux Mathematicorum 38. Cho a1 , a2 ,..., an , n ≥ 2 là n s th c sao cho a1 < a2 < ... < an . Ch ng minh r ng a1a2 + a2 a3 + ... + an a14 ≥ a2 a14 + a3a2 + ... + a1an . 4 4 4 4 39. [ Mircea Lascu ] Cho a, b, c là các s th c dương. Ch ng minh r ng b+c c +a a +b  a b c . + + ≥ 4  + +   a b c b + c c + a a + b  40. Cho a1 , a2 ,..., an là các s nguyên dương l n hơn 1. T n t i ít nh t m t trong các s 3 a1 a1 , a2 a3 ,..., an−1 an , an a1 nh hơn ho c b ng 3. Adapted after a well – known problem 41. [ Mircea Lascu, Marian Tetiva ] Cho x, y, z là các s th c dương th a mãn ñi u ki n xy + yz + zx + 2 xyz = 1 . Ch ng minh r ng 1 a) xyz ≤ , 8 3 b) x + y + z ≥ , 2 1 1 1 c) + + ≥ 4( x + y + z) , x y z 2 1 1 1 (2 z −1) d) + + − 4( x + y + z) ≥ , z = max { x, y, z } . x y z z (2 z +1) 42. [ Manlio Marangelli ] Cho x, y, z là các s th c dương. Ch ng minh r ng 3( x 2 y + y 2 z + z 2 x )( xy 2 + yz 2 + zx 2 ) ≥ xyz ( x + y + z ) . 3 43. [ Gabriel Dospinescu ] Cho a, b, c là các s th c dương th a mãn ñi u ki n max {a, b, c} − min {a, b, c} ≤ 1 Ch ng minh r ng 1 + a 3 + b3 + c 3 + 6abc ≥ 3a 2b + 3b 2 c + 3c 2 a . 44. [ Gabriel Dospinescu ] Cho a, b, c là các s th c dương. Ch ng minh r ng  a 2  b2  c2   1 1 1 27 + 2 + 2 + 2 +  ≥ 6 (a + b + c ) + +  .         bc    ca    ab  a b c     1 a2 45. Cho a0 = , a k+1 = ak + k . Ch ng minh r ng 2 n 1 1− < an < 1 . n TST Singapore 46. [ Călin Popa ] Cho a, b, c ∈ (0,1) th a mãn ñi u ki n ab + bc + ca = 1 . Ch ng minh r ng 6
  • 7. 500 Bài Toán B t ð ng Th c Ch n L c Cao Minh Quang a b c 3 1− a 2 1− b2 1− c 2   + + ≥  + + . 1 − a 2 1− b 2 1− c 2 4  a  b c    47. [ Titu Vàreescu, Gabriel Dospinescu ] Cho x, y, z ≤ 1 th a mãn ñi u ki n x + y + z = 1 . Ch ng minh r ng 1 1 1 27 2 + 2 + 2 ≤ . 1+ x 1+ y 1+ z 10 48. [ Gabriel Dospinescu ] Cho x + y + z = 1 . Ch ng minh r ng 2 2 2 (1− x) (1− y ) (1− z ) ≥ 215 xyz ( x + y )( y + z )( z + x) . 49. Cho x, y, z là các s th c dương th a mãn ñi u ki n xyz = x + y + z +2 . Ch ng minh r ng a) xy + yz + zx ≥ 2 ( x + y + z ) , 3 b) x+ y+ z≤ xyz . 2 50. Cho x, y, z là các s th c th a mãn ñi u ki n x 2 + y 2 + z 2 = 2 . Ch ng minh r ng x + y + z ≤ xyz + 2 . IMO Shortlist, 1987 51. [ Titu Vàreescu, Gabriel Dospinescu ] Cho x1 , x2 ,..., xn ∈ (0,1) và σ là m t hoán v c a {1, 2,..., n} . Ch ng minh r ng  n   ∑ xi   n      n 1 1 + i=1 .  1   ∑ 1− x ≥   ∑   1− x . x  .  n   i=1   i=1   i σ(i )  i       n 1 52. Cho x1 , x2 ,..., xn là các s th c dương th a mãn ñi u ki n ∑ 1+ x = 1 . Ch ng minh r ng i=1 i n n 1 ∑ xi ≥ (n −1) ∑ xi . i=1 i=1 Vojtech Jarnik n 53. [ Titu Vàreescu ] Cho n > 3 và a1 , a2 ,..., an là các s th c th a mãn ñi u ki n ∑a ≥ n i i=1 n và ∑a 2 i ≥ n 2 . Ch ng minh r ng i=1 max {a1 , a2 ,..., an } ≥ 2 . USAMO, 1999 54. [ Vasile Cirtoaje ] Cho a, b, c, d là các s th c dương. Ch ng minh r ng a −b b−c c − d d −a + + + ≥0. b+c c +d d +a a +b 55. Cho x, y là các s th c dương. Ch ng minh r ng x y + yx >1 . 7
  • 8. 500 Bài Toán B t ð ng Th c Ch n L c Cao Minh Quang France, 1996 56. Cho a, b, c là các s th c dương th a mãn ñi u ki n abc = 1 . Ch ng minh r ng (a + b)(b + c)(c + a ) ≥ 4 (a + b + c −1) . MOSP, 2001 57. Cho a, b, c là các s th c dương. Ch ng minh r ng (a 2 + b2 + c2 )(a + b − c)(b + c − a)(c + a − b) ≤ abc (ab + bc + ca) . 58. [ D.P.Mavlo ] Cho a, b, c là các s th c dương. Ch ng minh r ng 1 1 1 a b c (a + 1)(b +1)(c +1) 3+ a +b + c + + + + + + ≥ 3 . a b c b c a 1 + abc Kvant, 1988 59. [ Gabriel Dospinescu ] Cho x1 , x2 ,..., xn là các s th c dương th a mãn ñi u ki n x1 x2 ...xn = 1 . Ch ng minh r ng  n 1 n n n n .∏( x + 1) ≥ ∑ xi + ∑  . n n     i=1 i  i =1  x i=1 i 60. Cho a, b, c, d là các s th c dương th a mãn ñi u ki n a + b + c = 1 . Ch ng minh r ng 1 1 d    a 3 + b3 + c3 + abcd ≥ min  , +  .  4 9 27      Kvant, 1993 61. Cho a, b, c là các s th c dương. Ch ng minh r ng ∑ (1+ a ) (1 + b ) (a − c) (b − c) ≥ (1 + a )(1 + b )(1 + c )(a − b) (b − c) (c − a) . 2 2 2 2 2 2 2 2 2 2 2 2 AMM 62. [ Titu Vàreescu, Mircea Lascu ] Cho x, y, z là các s th c dương th a mãn ñi u ki n xyz = 1 và α ≥ 1. Ch ng minh r ng xα yα zα 3 + + ≥ . y+z z+x x+ y 2 63. Cho x1, x2 ,..., xn , y1, y2 ,..., yn ∈ ℝ th a mãn ñi u ki n x12 + x2 +... + xn = y12 + y2 +... + yn =1 . 2 2 2 2 Ch ng minh r ng  n  ( x1 y2 − x2 y1 ) ≤ 2 1− ∑ xi yi  .  2    i=1    Korea, 2001 64. [ Laurentiu Panaitopol ] Cho a1 , a2 ,..., an là các s nguyên dương khác nhau t ng ñôi m t. Ch ng minh r ng 2n + 1 a12 + a2 + ... + an ≥ 2 2 (a1 + a2 + ... + an ) . 3 TST Romania 65. [ Călin Popa ] Cho a, b, c là các s th c dương th a mãn ñi u ki n a + b + c = 1 . Ch ng minh r ng 8
  • 9. 500 Bài Toán B t ð ng Th c Ch n L c Cao Minh Quang b c c a a b 3 3 + + ≥ . a ( 3c + ab ) b ( 3a + bc ) c ( 3b + ca ) 4 66. [ Titu Vàreescu, Gabriel Dospinescu ] Cho a, b, c, d là các s th c th a mãn ñi u ki n (1 + a 2 )(1+ b2 )(1+ c 2 )(1 + d 2 ) = 16 . Ch ng minh r ng −3 ≤ ab + bc + cd + da + ac + bd − abcd ≤ 5 . 67. Cho a, b, c là các s th c dương. Ch ng minh r ng (a 2 + 2)(b2 + 2)(c2 + 2) ≥ 9(ab + bc + ca) . APMO, 2004 68. [ Vasile Cirtoale ] Cho x, y, z là các s th c th a mãn các ñi u ki n 0 < x ≤ y ≤ z, x + y + z = xyz + 2 . Ch ng minh r ng a) (1− xy )(1− yz )(1− zx) ≥ 0 , 32 b) x 2 y ≤ 1, x 3 y 2 ≤ . 27 69. [ Titu Vàreescu ] Cho a, b, c là các s th c dương th a mãn ñi u ki n a + b + c ≥ abc . Ch ng minh r ng ít nh t m t trong ba b t ñ ng th c sau ñây là ñúng 2 3 6 2 3 6 2 3 6 + + ≥ 6, + + ≥ 6, + + ≥ 6 . a b c b c a c a b TST 2001, USA 70. [ Gabriel Dospinescu, Marian Tetiva ] Cho x, y, z là các s th c dương th a mãn ñi u ki n x + y + z = xyz . Ch ng minh r ng ( x −1)( y −1)( z −1) ≤ 6 3 −10 . 71. [ Marian Tetiva ] Cho a, b, c là các s th c dương. Ch ng minh r ng 2 2 2 a3 − b3 b3 − c3 c 3 − a3 (a − b) + (b − c ) + (c − a ) + + ≤ . a +b b+c c+a 4 Moldova TST, 2004 72. [ Titu Vàreescu ] Cho a, b, c là các s th c dương. Ch ng minh r ng (a5 − a 2 + 3)(b5 − b2 + 3)(c5 − c 2 + 3) ≥ (a + b + c)3 . USAMO, 2004 73. [ Gabriel Dospinescu ] Cho x1 , x2 ,..., xn > 0, n > 2 th a mãn ñi u ki n  n  n 1    x  ∑ k   ∑  = n 2 + 1 .   k =1  k =1 xk     Ch ng minh r ng  n 2  n 1    2  x   ∑ k  ∑ 2  > n + 4 + 2 .   k =1  k =1 xk   n (n −1)   74. [ Gabriel Dospinescu, Mircea Lascu, Marian Tetiva ] Cho a, b, c là các s th c dương. Ch ng minh r ng 9
  • 10. 500 Bài Toán B t ð ng Th c Ch n L c Cao Minh Quang a 2 + b 2 + c 2 + 2abc + 3 ≥ (1 + a)(1 + b)(1 + c) . 75. [ Titu Vàreescu, Zuming Feng ] Cho a, b, c là các s th c dương. Ch ng minh r ng 2 2 2 ( 2a + b + c ) (2b + a + c) (2c + b + c) 2 + 2 2 + 2 2 ≤8. 2a + (b + c) 2 2b + (a + c) 2c + (a + b) USAMO, 2003 76. Cho x, y là các s th c dương và m, n là các s nguyên dương. Ch ng minh r ng (n −1)(m −1)( x m+n + y m+n ) + (m + n −1)( x m y n + x n y m ) ≥ mn ( x m+n−1 y + y m+n−1 x) . Austrian – Polish Competition, 1995 77. Cho a, b, c, d , e là các s th c dương th a mãn ñi u ki n abcde = 1 . Ch ng minh r ng a + abc b + bcd c + cde d + dea e + eab 10 + + + + ≥ . 1 + ab + abcd 1 + bc + bcde 1 + cd + cdea 1 + de + deab 1 + ea + eabc 3 Crux Mathematicorum  π 78. [ Titu Vàreescu ] Cho a, b, c ∈ 0,  . Ch ng minh r ng  2     sin a.sin (a − b).sin (a − c ) sin b.sin (b − c ).sin (b − a ) sin c.sin (c − a ).sin (c − b) + + ≥0. sin (b + c ) sin (c + a ) sin (a + b) TST 2003, USA 79. Cho a, b, c là các s th c dương. Ch ng minh r ng a 4 + b4 + c 4 + a 2b 2 + b 2c 2 + c 2 a 2 ≥ a 3b + b3c + c 3a + ab3 + bc3 + ca 3 . KMO Summer Program Test, 2001 80. [ Gabriel Dospinescu, Mircea Lascu ] Cho a1 , a2 ,..., an > 0, n > 2 th a mãn ñi u ki n a1a2 ...an = 1 . Hãy tìm h ng s kn nh nh t sao cho a1a2 a2 a3 an a1 + + ... + ≤ kn . (a2 1 + a2 )(a + a1 ) 2 2 (a 2 2 + a3 )(a + a2 ) 2 3 (a 2 n + a1 )(a12 + an ) 81. [ Vasile Cirtoaje ] Cho a, b, c, x, y, z là các s th c dương. Ch ng minh r ng 2 ax + by + cz + (a 2 + b2 + c 2 )( x 2 + y 2 + z 2 ) ≥ 3 (a + b + c)( x + y + z ) . Kvant, 1989 82. [ Vasile Cirtoaje ] Cho a, b, c là ñ dài ba c nh c a m t tam giác. Ch ng minh r ng a b c  b c a 3 + + −1 ≥ 2  + +  .       b c a   a b c  83. [ Walther Janous ] Cho x1 , x2 ,..., xn > 0, n > 2 th a mãn ñi u ki n x1 + x2 + ... + xn = 1 . Ch ng minh r ng   n   1 + 1  ≥  n − xi  . n ∏ x  ∏ 1 − x   i=1     i=1      i i Crux Mathematicorum 10
  • 11. 500 Bài Toán B t ð ng Th c Ch n L c Cao Minh Quang 84. [ Vasile Cirtoaje, Gheoghe Eckstein ] Cho x1 , x2 ,..., xn là các s th c dương th a mãn ñi u ki n x1 x2 ...xn = 1 . Ch ng minh r ng 1 1 1 + + ... + ≤1 . n −1 + x1 n −1 + x2 n −1 + xn TST 1999, Romania 85. [ Titu Vàreescu ] Cho a, b, c là các s th c không âm th a ñi u ki n a2 +b2 +c2 +abc = 4 . Ch ng minh r ng 0 ≤ ab + bc + ca − abc ≤ 2 . USAMO, 2001 86. [ Titu Vàreescu ] Cho a, b, c là các s th c dương. Ch ng minh r ng a +b +c 3 {( ) ( ) ( ) }. 2 2 2 − abc ≤ max a− b , b− c , c− a 3 TST 2000, USA 87. [ Kiran Kedlaya ] Cho a, b, c là các s th c dương. Ch ng minh r ng a + ab + 3 abc 3 a + b a + b + c ≤ a. . . 3 2 3 88. Tìm h ng s k l n nh t sao cho v i b t kì s nguyên dương n không chính phương, ta có (1+ n ) sin (π n ) > k . Vietnamese IMO Training Camp, 1995 3 89. [ Tr n Nam Dũng ] Cho x, y, z là các s th c dương th a ñi u ki n ( x + y + z ) = 32 xyz . Tìm giá tr l n nh t và giá tr nh nh t c a bi u th c x4 + y4 + z 4 4 . (x + y + z) Vietnam, 2004 90. [ George Tsintifas ] Cho a, b, c, d là các s th c dương. Ch ng minh r ng 3 3 3 3 4 (a + b) (b + c) (c + d ) (d + a) ≥ 16a 2b2 c 2 d 2 (a + b + c + d ) . Crux Mathematicorum 91. [ Titu Vàreescu, Gabriel Dospinescu ] Cho a, b, c là các s th c không âm th a mãn ñi u ki n a + b + c = 1 và n là s nguyên dương. Tìm giá tr l n nh t c a bi u th c (ab) (bc) (ca) n n n + + . 1− ab 1− bc 1− ca 92. Cho a, b, c là các s th c dương. Ch ng minh r ng 1 1 1 3 + + ≥ . a (1 + b) b (1 + c ) c (1 + a ) 3 abc 1 + 3 abc ( ) 93. [Tr n Nam Dũng ] Cho a, b, c là các s th c dương th a mãn ñi u ki n a 2 + b2 + c 2 = 9 . Ch ng minh r ng 11
  • 12. 500 Bài Toán B t ð ng Th c Ch n L c Cao Minh Quang 2 (a + b + c) − abc ≤ 10 . Vietnam, 2002 94. [ Vasile Cirtoaje ] Cho a, b, c là các s th c dương. Ch ng minh r ng          a + 1 −1b + 1 −1 + b + 1 −1c + 1 −1 + c + 1 −1a + 1 −1 ≥ 3 .            b    c   c    a   a  b   95. [ Gabriel Dospinescu ] Cho n là s nguyên l n hơn 2. Tìm s th c l n nh t mn và s th c nh nh t M n sao cho v i các s th c dương b t kì x1 , x2 ,..., xn (xem xn = x0 , xn+1 = x1 ), ta có n xi mn ≤ ∑ ≤ Mn . i=1 xi−1 + 2 (n −1) xi + xi +1 96. [ Vasile Cirtoaje ] Cho x, y, z là các s th c dương. Ch ng minh r ng 1 1 1 9 2 2 + 2 2 + 2 2 ≥ 2 . x + xy + y y + yz + z z + zx + x (x + y + z) Gazeta Matematică 97. [ Vasile Cirtoaje ] Cho a, b, c, d là các s th c dương. Ch ng minh r ng 2 (a3 +1)(b3 +1)(c 3 + 1)(d 3 +1) ≥ (1 + abcd )(1 + a 2 )(1 + b 2 )(1 + c 2 )(1 + d 2 ) . Gazeta Matematică 98. Cho a, b, c là các s th c dương. Ch ng minh r ng 4 4 4 4 (a + b) + (b + c) + (c + a) ≥ 4 7 (a + b 4 + c 4 ) . Vietnam TST, 1996 99. Cho a, b, c là các s th c dương th a mãn ñi u ki n abc = 1 . Ch ng minh r ng 1 1 1 1 1 1 + + ≤ + + . 1+ a + b 1+ b + c 1+ c + a 2 + a 2 + b 2 + c Bulgaria, 1997 100. [Tr n Nam Dũng ] Cho a, b, c là các s th c dương th a 21ab + 2bc + 8ca ≤ 12 . Tìm giá tr nh nh t c a bi u th c 1 2 3 + + . a b c Vietnam, 2001 101. [ Titu Vàreescu, Gabriel Dospinescu ] Cho a, b, c, x, y, z là các s th c dương th a mãn ñi u ki n xy + yz + zx = 3 . Ch ng minh r ng a b c ( y + z)+ ( z + x) + ( x + y) ≥ 3 . b+c c+a a +b 102. Cho a, b, c là các s th c dương. Ch ng minh r ng 2 2 2 (b + c − a ) (c + a − b) (a + b − c) 3 2 + 2 + 2 ≥ . (b + c) + a 2 (c + a) + b 2 (a + b) + c 2 5 Japan, 1997 12
  • 13. 500 Bài Toán B t ð ng Th c Ch n L c Cao Minh Quang 103. [ Vasile Cirtoaje, Gabriel Dospinescu ] Cho a1 , a2 ,..., an ≥ 0, an = min {a1 , a2 ,..., an } . Ch ng minh r ng  a + a2 + ... + an−1 n a1n + a2 + ... + an − na1a2 ...an ≥ (n −1) 1 n n  − an  .     n −1  104. [ Turkervici ] Cho x, y , z , t là các s th c dương. Ch ng minh r ng x 4 + y 4 + z 4 + t 4 + 2 xyzt ≥ x 2 y 2 + y 2 z 2 + z 2t 2 + x 2 z 2 + y 2t 2 . Kvant 105. Cho a1 , a2 ,..., an là các s th c dương. Ch ng minh r ng  n 2 n  a ≤ ∑ i  ij  ∑ i + j −1ai a j .  i=1  i , j=1   106. Cho a1 , a2 ,..., an , b1 , b2 ,..., bn ∈ (1001, 2002) sao cho a12 + a2 + ... + an = b12 + b2 + ... + bn . 2 2 2 2 Ch ng minh r ng a13 a2 3 a 3 17 + + ... + n ≤ (a12 + a2 + ... + an ) . 2 2 b1 b2 bn 10 TST Singapore 107. [ Titu Vàreescu, Gabriel Dospinescu ] Cho a, b, c là các s th c dương th a mãn ñi u ki n a + b + c = 1 . Ch ng minh r ng (a 2 + b2 )(b2 + c 2 )(c 2 + a 2 ) ≥ 8(a 2b2 + b2c 2 + c 2 a 2 ) 2 . 108. [ Vasile Cirtoaje ] Cho a, b, c, d là các s th c dương th a mãn ñi u ki n abcd = 1 . Ch ng minh r ng 1 1 1 1 2 + 2 + 2 + 2 ≥1. (1 + a ) (1 + b) (1 + c) (1 + d ) Gazeta Matematică 109. [ Vasile Cirtoaje ] Cho a, b, c là các s th c dương. Ch ng minh r ng a2 b2 c2 a b c 2 2 + 2 2 + 2 2 ≥ + + . b +c c +a a +b b+c c +a a +b Gazeta Matematică 110. [ Gabriel Dospinescu ] Cho n s th c a1 , a2 ,..., an . Ch ng minh r ng 2    a ≤ 2  1≤∑ n ( i ∑ i    a + ... + a j ) . i∈ℕ*    i≤ j≤ TST 2004, Romania 111. [Tr n Nam Dũng ] Cho x1 , x2 ,..., xn ∈ [−1,1] th a mãn ñi u ki n x1 + x2 + ... + xn = 0 . 3 3 3 Tìm giá tr l n nh t c a bi u th c x1 + x2 + ... + xn . 112. [ Gabriel Dospinescu, Călin Popa ] Cho n s th c a1 , a2 ,..., an , n ≥ 2 th a mãn ñi u ki n a1a2 ...an = 1 . Ch ng minh r ng 13
  • 14. 500 Bài Toán B t ð ng Th c Ch n L c Cao Minh Quang 2n n a12 + a2 + ... + an − n ≥ 2 2 n −1 (a1 + a2 + ... + an − n) . n −1 113. [ Vasile Cirtoaje ] Cho a, b, c là các s th c dương. Ch ng minh r ng 2a 2b 2c + + ≤ 3. a +b b+c c+a Gazeta Matematică 114. Cho x, y, z là các s th c dương. Ch ng minh r ng  1 1 1  9 ( xy + yz + zx)  2 + 2 + 2 ≥ .  ( x + y ) ( y + z) ( z + x)  4 Iran, 1996 115. [ Cao Minh Quang ] Cho x1 , x2 ,..., xn là các s th c dương th a mãn ñi u ki n n ∏(3x +1) ≤ 2 i=1 i n . Ch ng minh r ng n 1 n ∑ 6 x +1 ≥ 3 . i=1 i 116. [ Suranyi ] Cho a1 , a2 ,..., an là các s th c dương. Ch ng minh r ng (n −1)(a1n + a2 + ... + an ) + na1a2 ...an ≥ (a1 + a2 + ... + an )(a1n−1 + a2 −1 + ... + ann−1 ) . n n n Miklos Schweitzer Competition 117. [ Gabriel Dospinescu ] Cho x1 , x2 ,..., xn > 0 th a mãn ñi u ki n x1 x2 ...xn = 1 . Ch ng minh r ng n ∑ (x − x ) ≥ ∑ x 2 2 i j i −n . 1≤i≤ j≤n i =1 A generazation of Tukervici’s Inequality 1 118. [ Vasile Cirtoaje ] Cho a1 , a2 ,..., an < và a1 + a2 + ... + an = 1, n > 2 . Tìm giá tr n −1 nh nh t c a bi u th c n a1a2 ...an ∑ 1−(n −1) ai . i=1 119. [ Vasile Cirtoaje ] Cho a1 , a2 ,..., an ∈ [0,1) th a mãn ñi u ki n a12 + a2 + ... + an 2 2 3 a= ≥ . n 3 Ch ng minh r ng a1 a a na 2 + 2 2 + ... + n 2 ≥ . 1− a1 1− a2 1− an 1− a 2 120. [ Vasile Cirtoaje, Mircea Lascu ] Cho a, b, c, x, y, z là các s th c dương th a mãn ñi u ki n 14
  • 15. 500 Bài Toán B t ð ng Th c Ch n L c Cao Minh Quang (a + b + c)( x + y + z ) = (a 2 + b 2 + c 2 )( x 2 + y 2 + z 2 ) = 4 . Ch ng minh r ng 1 abcxyz < . 36 121. [ Gabriel Dospinescu ] Cho x1 , x2 ,..., xn > 0, n > 2 th a mãn ñi u ki n x1 x2 ...xn = 1 . Tìm h ng s kn nh nh t sao cho 1 1 1 + + ... + ≤ n −1 . 1 + kn x1 1 + kn x2 1 + kn xn Mathlinks Contest 122. [ Vasile Cirtoaje, Gabriel Dospinescu ] Cho x1 , x2 ,..., xn > 0, n > 2 th a mãn ñi u ki n x12 + x2 + ... + xn = 1 . Tìm h ng s kn l n nh t sao cho 2 2 (1− x1 )(1− x2 )...(1− xn ) ≥ kn x1 x2 ...xn . 123. Cho a, b, c là các s th c dương th a mãn ñi u ki n abc = 1 . Ch ng minh r ng 1 1 1 3 + 3 + 3 ≥ . a (b + c ) b (c + a ) c (a + b) 2 3 IMO, 1995 124. Cho a, b, c là các s th c dương th a mãn ñi u ki n abc = 1 . Ch ng minh r ng ab bc ca + 5 + 5 ≤ 1. a + b + ab b + c + bc c + a 5 + ca 5 5 5 IMO Shortlist, 1996 125. Cho a, b, c là các s th c dương th a mãn ñi u ki n abc = 1 . Ch ng minh r ng 1 + ab 2 1 + bc 2 1 + ca 2 18 3 + 3 + 3 ≥ 3 . c a b a + b3 + c3 Hong Kong, 2000 126. Cho a, b, c là các s th c dương th a mãn ñi u ki n abc = 1 . Ch ng minh r ng 1 1 1 1 2 + 2 + 2 ≤ . (a +1) + b + 1 (b +1) + c + 1 (c +1) + a + 1 2 2 2 2 127. Cho a, b, c là các s th c dương th a mãn ñi u ki n abc = 1 . Ch ng minh r ng     a −1 + 1 b −1 + 1 c −1 + 1  ≤ 1 .       b   c   a IMO, 2000 128. Cho a, b, c là các s th c dương th a mãn ñi u ki n abc = 1 . Ch ng minh r ng a3 b3 c3 3 + + ≥ . (1 + b)(1 + c) (1 + a)(1 + c) (1 + a )(1 + b) 4 IMO Shortlist, 1998 129. Cho a, b, c là các s th c dương th a mãn ñi u ki n a + b + c = 1 . Ch ng minh r ng 15
  • 16. 500 Bài Toán B t ð ng Th c Ch n L c Cao Minh Quang ab bc ca 1 + + ≤ . 1+ c 1+ a 1+ b 4 130. Cho a, b, c là các s th c dương th a mãn ñi u ki n a + b + c = 1 . Ch ng minh r ng a 2 + b 2 + c 2 + 2 3abc ≤ 1 . Poland, 1999 131. Cho a, b, c là các s th c dương th a mãn ñi u ki n a 2 + b 2 + c 2 = 1 . Ch ng minh r ng 1 a +b+c + ≥4 3. abc Macedonia, 1999 132. Cho a, b, c là các s th c dương th a mãn ñi u ki n a + b + c = 1 . Ch ng minh r ng ab + c + bc + a + ca + b ≥ 1 + ab + bc + ca . 133. Cho a, b, c là các s th c dương th a mãn ñi u ki n a + b + c = 1 . Ch ng minh r ng (1 + a)(1 + b)(1 + c) ≥ 8 (1− a )(1− b)(1− c) . Russia, 1991 134. Cho a, b là các s th c dương th a mãn ñi u ki n a + b = 1 . Ch ng minh r ng a2 b2 1 + ≥ . a +1 b +1 3 Hungary, 1996 135. Cho các s th c x, y . Ch ng minh r ng 2 3( x + y + 1) + 1 ≥ 3 xy . Columbia, 2001 136. Cho a, b, c là các s th c dương. Ch ng minh r ng  1 1 a b 2 (a + b) +  ≥ 3 + 3 . a b  3    b a Czech and Slovakia, 2000 137. Cho a, b, c ≥ 1 . Ch ng minh r ng a −1 + b −1 + c −1 ≤ c (ab + 1) . Hong Kong, 1998 138. Cho x, y, z là các s th c dương th a mãn ñi u ki n x + y + z = xyz . Ch ng minh r ng 1 1 1 3 + + ≤ . 1+ x 2 1+ y 2 1+ z 2 2 Korea, 1998 139. Cho a, b, c là các s th c dương. Ch ng minh r ng a b c + + ≥1 . 2 2 2 a + 8bc b + 8ca c + 8ab IMO, 2001 16
  • 17. 500 Bài Toán B t ð ng Th c Ch n L c Cao Minh Quang 140. Cho a, b, c, d là các s th c dương. Ch ng minh r ng a b c d 2 + + + ≥ . b + 2c + 3d c + 2d + 3a d + a + 3b a + 2b + 3c 3 IMO Shortlist, 1993 141. Cho a, b, c, d là các s th c dương th a mãn ñi u ki n ab + bc + cd + da = 1 . Ch ng minh r ng a3 b3 c3 d3 1 + + + ≥ . b+c +d c +d +a d +a +b a +b+c 3 IMO Shortlist, 1990 142. Cho a, b, c là các s th c dương. Ch ng minh r ng a2 b2 c2 bc ca ab 2 + 2 + 2 ≥1 ≥ 2 + 2 + 2 . a + 2bc b + 2ca c + 2ab a + 2bc b + 2ca c + 2ab Romania, 1997 143. Cho a, b, c là các s th c dương. Ch ng minh r ng a 3 b3 c3 + + ≥ a +b +c . bc ca ab Canada, 2002 144. Cho a, b, c là các s th c dương. Ch ng minh r ng 1 1 1 1 3 3 + 3 3 + 3 3 ≤ . a + b + abc b + c + abc c + a + abc abc USA, 1997 145. Cho a, b, c là các s th c dương th a mãn ñi u ki n a2 + b2 + c2 = 3 . Ch ng minh r ng 1 1 1 3 + + ≥ . 1 + ab 1 + bc 1 + ca 2 Belarus, 1999 146. Cho a, b, c là các s th c dương. Ch ng minh r ng a b c a +b b + c + + ≥ + +1. b c a b+c a +b Belarus, 1998 3 147. Cho a, b, c ≥ − , a + b + c = 1 . Ch ng minh r ng 4 a b c 9 2 + 2 + 2 ≤ . a + 1 b + 1 c + 1 10 Poland, 1996 148. Cho x, y, z là các s th c dương th a mãn ñi u ki n xyz = 1 . Ch ng minh r ng x9 + y 9 y9 + z9 z 9 + x9 + 6 + 6 ≥2. x6 + x3 y 3 + y 6 y + y 3 z 3 + z 6 z + z 3 z 3 + x 6 Roamania, 1997 149. Cho x ≥ y ≥ z > 0 . Ch ng minh r ng 17
  • 18. 500 Bài Toán B t ð ng Th c Ch n L c Cao Minh Quang x2 y y2 z z 2 x + + ≥ x2 + y2 + z 2 . z x y Vietnam, 1991 150. Cho a ≥ b ≥ c > 0 . Ch ng minh r ng a 2 − b 2 c 2 − b2 a 2 − c 2 + + ≥ 3a − 4b + c . c a b Ukraine, 1992 151. Cho x, y, z là các s th c dương. Ch ng minh r ng ( xyz x + y + z + x 2 + y 2 + z 2 ) ≤ 3+ 3 . (x 2 + y + z )( xy + yz + zx ) 2 2 9 Hong Kong, 1997 152. Cho a1 , a2 , ..., an > 0 và a1 + a2 + ... + an < 1 . Ch ng minh r ng a1a2 ...an (1− a1 − a2 − ... − an ) 1 ≤ n+1 . (a1 + a2 + ... + an )(1− a1 )(1− a2 )...(1− an ) n IMO Shortlist, 1998 153. Cho hai s th c a, b , a ≠ 0 . Ch ng minh r ng 1 b a 2 + b2 + + ≥ 3. a2 a Austria, 2000 154. Cho a1 , a2 , ..., an > 0 . Ch ng minh r ng a12 a2 2 a2 a2 + + ... + n−1 + n ≥ a1 + a2 + ... + an . a2 a3 an a1 China, 1984 155. Cho x, y, z là các s th c dương th a mãn ñi u ki n xyz = 1 . Ch ng minh r ng x 2 + y 2 + z 2 + x + y + z ≥ 2 ( xy + yz + zx) . Russia, 2000 156. Cho x, y, z là các s th c dương th a mãn ñi u ki n xyz ≥ xy + yz + zx . Ch ng minh r ng xyz ≥ 3( x + y + z ) . India, 2001 1 1 1 157. Cho x, y, z > 1 và + + = 2 . Ch ng minh r ng x y z x + y + z ≥ x −1 + y −1 + z −1 . IMO, 1992 158. Cho a, b, c là các s th c dương th a mãn ñi u ki n ab +bc +ca =1. Ch ng minh r ng 1 1 1 1 3 + 6b + 3 + 6c + 3 + 6a ≤ . a b c abc 18
  • 19. 500 Bài Toán B t ð ng Th c Ch n L c Cao Minh Quang IMO Shortlist, 2004 159. Cho x ≥ 2, y ≥ 2, z ≥ 2 . Ch ng minh r ng ( x3 + y )( y 3 + z )( z 3 + x) ≥ 125 xyz . Saint Petersburg, 1997 160. Cho a, b, c, d là các s th c dương th a mãn ñi u ki n c 2 + d 2 = (a 2 + b 2 ) . Ch ng 3 minh r ng a 3 b3 + ≥ 1. c d Singapore, 2000 161. Cho a, b, c là các s th c dương. Ch ng minh r ng a b c + + ≥1. b + 2c c + 2a a + 2b Czech – Slovak Match, 1999 162. Cho a, b, c là các s th c dương. Ch ng minh r ng ab bc ca a b c + + ≥ + + . c (c + a) a (a + b) b (b + c) c + a b + a c + b Moldova, 1999 163. Cho a, b, c, d là các s th c dương. Ch ng minh r ng a +c b+d c +a d +b + + + ≥ 4. a+b b+c c +d d +a Baltic way, 1995 164. Cho x, y, u , v là các s th c dương. Ch ng minh r ng xy + xu + uy + uv xy uv ≥ + . x + y +u +v x+ y u +v Poland, 1993 165. Cho a, b, c là các s th c dương. Ch ng minh r ng  a  b  c    1 + 1 + 1 +  ≥ 2 1 + a + b + c  .   b  c  a             3 abc   APMO, 1998 166. Cho x, y, z là các s th c không âm th a mãn ñi u ki n x + y + z =1. Ch ng minh r ng 4 x2 y + y 2 z + z 2 x ≤ . 27 Canada, 1999 167. Cho a, b, c, d , e, f là các s th c dương th a mãn ñi u ki n 1 a + b + c + d + e + f = 1, ace + bdf ≥ . 108 Ch ng minh r ng 19
  • 20. 500 Bài Toán B t ð ng Th c Ch n L c Cao Minh Quang 1 abc + bcd + cde + def + efa + fab ≤ . 36 Poland, 1998 168. Cho a, b, c ∈ [0,1] . Ch ng minh r ng a 2 + b 2 + c 2 ≤ a 2b + b 2 c + c 2 a + 1 . Italy, 1993 169. Cho a, b, c ≥ 0, a + b + c ≥ abc . Ch ng minh r ng a 2 + b 2 + c 2 ≥ abc . Ireland, 1997 170. Cho a, b, c ≥ 0, a + b + c ≥ abc . Ch ng minh r ng a 2 + b 2 + c 2 ≥ 3abc . BMO, 2001 171. Cho x, y, z là các s th c dương th a mãn ñi u ki n x + y + z = xyz . Ch ng minh r ng xy + yz + zx ≥ 9 ( x + y + z ) . Belarus, 1996 172. Cho x1 , x2 , x3 , x4 là các s th c dương th a mãn ñi u ki n x1 x2 x3 x4 = 1 . Ch ng minh r ng   1 1 1 1  x13 + x2 + x3 + x4 ≥ max  x1 + x2 + x3 + x4 , + + +  . 3 3 3      x1 x2 x3 x4    Iran, 1997 173. Cho a, b, c, x, y, z là các s th c dương. Ch ng minh r ng 3 a 3 b3 c 3 (a + b + c ) + + ≥ . x y z 3( x + y + z ) Belarus TST, 2000 174. Cho a, b, c, d là các s th c dương th a mãn ñi u ki n 1 1 1 1 4 + 4 + 4 + =1. 1+ a 1+ b 1+ c 1+ d 4 Ch ng minh r ng abcd ≥ 3 . Latvia, 2002 175. Cho x, y, z > 1 . Ch ng minh r ng 2 +2 yz 2 + 2 zx 2 +2 xy xy + yz + zx xx yy zz ≥ ( xyz ) . Proposed for 1999 USAMO 176. Cho c ≥ b ≥ a ≥ 0 . Ch ng minh r ng (a + 3b)(b + 4c)(c + 2a) ≥ 60abc . Turkey, 1999 20
  • 21. 500 Bài Toán B t ð ng Th c Ch n L c Cao Minh Quang 177. Cho x, y, z là các s th c dương. Ch ng minh r ng x 2 + y 2 + z 2 ≥ 2 ( xy + yz ) . Macedonia, 2000 178. Cho các s th c a, b, c th a mãn ñi u ki n a 2 + b 2 + c 2 = 1 . Ch ng minh r ng a2 b2 c2 3 + + ≥ . 1 + 2bc 1 + 2ca 1 + 2ab 5 Bosnia and Hercegovina, 2002 179. Cho a, b, c là các s th c dương th a mãn ñi u ki n abc ≥ 1 . Ch ng minh r ng 1 1 1 4 4 + 4 4 + 4 ≤1. a+b +c a +b+c a + b4 + c Korea, 1999 180. Cho a > b > c > 0, x > y > z > 0 . Ch ng minh r ng a 2 x2 b2 y 2 c2 z 2 3 + + ≥ . (by + cz )(bz + cy ) (cz + ax)(cx + az ) (ax + by )(ay + bx) 4 Korea, 2000 181. Cho a, b, c là các s th c không âm th a mãn ñi u ki n a + b + c = 3 . Ch ng minh r ng a b c 3 + + ≥ . b2 +1 c 2 +1 a 2 + 1 2 Mediterranean, 2003 182. Cho a, b, c là các s th c dương. Ch ng minh r ng a b c + + ≤1. 2a + b 2b + c 2c + a Moldova, 2002 183. Cho α, β , x1 , x2 ,..., xn > 0, x1 + x2 + ... + xn = 1 . Ch ng minh r ng 3 x13 3 x2 xn 1 + + ... + ≥ . α x1 + β x2 α x2 + β x3 α xn + β x1 n (α + β ) Moldova TST, 2002 184. Cho a là m t s th c dương, x1 , x2 ,..., xn > 0, x1 + x2 + ... + xn = 1 . Ch ng minh r ng a x1−x2 a x2 −x3 a xn −x1 n2 + + ... + ≥ . x1 + x2 x2 + x3 xn + x1 2 Serbia, 1998 185. Cho x, y ∈ [ 0,1] . Ch ng minh r ng 1 1 2 + ≤ . 1+ x 2 1+ y 2 1 + xy Russia, 2000 21
  • 22. 500 Bài Toán B t ð ng Th c Ch n L c Cao Minh Quang 1 1 1 186. Cho x, y , z > 0, xyz = 1, + + > x + y + z, k ∈ N * . Ch ng minh r ng x y z 1 1 1 k + k + k > xk + y k + z k . x y z Russia, 1999 187. Cho xn ≥ xn−1 ≥ xn−2 ≥ ... ≥ x1 > 0, n ≥ 3 . Ch ng minh r ng xn x1 x1 x2 x x + + ... + n−1 n ≥ x1 + x2 + ... + xn . x2 x3 x1 Saint Petersburg, 2000 188. Cho x1 ,..., x6 ∈ [ 0,1] . Ch ng minh r ng 3 x13 x23 x6 3 5 5 5 + 5 5 5 + ... + 5 5 5 ≤ . x2 + x3 + ... + x6 + 5 x3 + x4 + ... + x1 + 5 x1 + x2 + ... + x5 + 5 5 Ukraine, 1999 189. Cho a1 , a2 ,..., an > 0 . Ch ng minh r ng (a13 +1)(a23 +1)...(an3 +1) ≥ (a12 a2 +1)(a22 a3 +1)...(an2a1 +1) . Czech – Slovak – Polish Match 2001 190. Cho a, b, c là các s th c dương th a mãn ñi u ki n a + b + c = 1 . Ch ng minh r ng a. 3 1 + b − c + b. 3 1 + c − a + c. 3 1 + a − b ≤ 1 . Japan, 2005 191. Cho a, b, c là các s th c dương. Ch ng minh r ng  a b c 2     + +  ≥ (a + b + c ) 1 + 1 + 1  .    b c a   a b c   Iran, 2005 192. Cho a, b, c, d là các s th c dương. Ch ng minh r ng 1 1 1 1 a+b+c +d 3 + 3+ 3+ 3≥ . a b c d abcd Austria, 2005 193. Cho a, b, c ∈ [0,1] . Ch ng minh r ng a b c + + ≤2. bc + 1 ca + 1 ab + 1 Poland, 2005 194. Cho a, b, c là các s th c dương th a mãn ñi u ki n a + b + c = 1 . Ch ng minh r ng 1 a b +b c +c a ≤ . 3 Bosnia and Hercegovina, 2005 195. Cho a, b, c là các s th c dương th a mãn ñi u ki n a + b + c = 1 . Ch ng minh r ng 22
  • 23. 500 Bài Toán B t ð ng Th c Ch n L c Cao Minh Quang  b c a  1+ a 1+ b 1+ c 2 + +  ≥    + + .   a b c  1 − a 1− b 1− c Germany, 2005 196. Cho a, b, c là các s th c dương. Ch ng minh r ng 2 a2 b2 c 2 4 ( a − b) + + ≥ a +b+c + . b c a a +b+c Balkan, 2005 197. Cho a, b, c là các s th c dương th a mãn ñi u ki n abc = 8 . Ch ng minh r ng a2 b2 c2 4 + + ≥ . (a 3 + 1)(b +1) 3 (b 3 + 1)(c + 1) 3 (c +1)(a +1) 3 3 3 APMO, 2005 198. Cho a, b, c là các s th c dương th a mãn ñi u ki n abc = 1 . Ch ng minh r ng a b c 2 + 2 + 2 ≤1 . a +2 b +2 c +2 Baltic way, 2005 199. Cho x, y, z là các s th c dương th a mãn ñi u ki n xyz ≥ 1 . Ch ng minh r ng x5 − x 2 y5 − y 2 z5 − z 2 + 5 + 5 ≥0. x5 + y 2 + z 2 y + z 2 + x 2 z + x 2 + y 3 IMO, 2005 200. Cho a, b, c là các s th c dương. Ch ng minh r ng  2      a + b + 3 b 2 + a + 3  ≥ 2a + 1 2b + 1           4  4    2  2 Belarusian, 2005 1 1 1 201. Cho a, b, c là các s th c dương th a mãn ñi u ki n + + = 1 . Ch ng minh r ng a b c (a −1)(b −1)(c −1) ≥ 8 Croatia, 2005 202. Cho x là s th c dương. Ch ng minh r ng (2 x) n n +1 1+ x ≥ n−1 . (1 + x) Russia, 2005 203. Cho a, b, c là các s th c dương th a mãn ñi u ki n abc ≥ 1 . Ch ng minh r ng 1 1 1 + + ≤ 1. 1+ a + b 1+ b + c 1+ c + a Romania, 2005 204. Cho a, b, c là các s th c dương th a mãn ñi u ki n abc = 1 . Ch ng minh r ng 23
  • 24. 500 Bài Toán B t ð ng Th c Ch n L c Cao Minh Quang a a a 3 + + ≥ . (a +1)(b +1) (b +1)(c +1) (c +1)(a + 1) 4 Czech and Slovak, 2005 1 205. Cho a, b, c là các s th c không âm th a mãn ñi u ki n ab + bc + ca = . Ch ng minh 3 r ng 1 1 1 2 + 2 + 2 ≤3. a − bc + 1 b − ca + 1 c − ab + 1 China, 2005 206. Cho a, b, c là các s th c dương th a mãn ñi u ki n a + b + c = 1 . Ch ng minh r ng 2 ab (1− c ) + bc (1− a ) + ca (1− b) ≤ . 3 Republic of Srpska, 2005 207. Cho a, b, c là các s th c dương. Ch ng minh r ng a b c 3 + + ≥ (a + b + c) . b+c c+a a +b 2 Serbia and Montenegro, 2005 208. Cho a, b, c là các s th c dương th a mãn ñi u ki n a 4 + b 4 + c 4 = 3 . Ch ng minh r ng 1 1 1 + + ≤1 . 4 − ab 4 − bc 4 − ca Moldova, 2005 209. Cho a, b, c là các s th c dương th a mãn ñi u ki n ab +bc +ca =1. Ch ng minh r ng 3 1 3 3. 3 + 6 (a + b + c) ≤ . abc abc Slovenia TST, 2005 210. Cho a, b, c là các s th c dương th a mãn ñi u ki n a, b, c ≥ 1 . Ch ng minh r ng 1 1 1 (2 + abc) + +  ≥ 9 . a b c     211. [ Huỳnh T n Châu ] Cho x, y, z là các s th c dương th a mãn ñi u ki n xy xy + yz yz + zx zx = 1 . Ch ng minh r ng x6 y6 z6 1 3 3 + 3 3 + 3 3 ≥ . x +y y +z z +x 2 212. [ ð ng Thanh H i ] Cho x là m t s th c b t kì. Ch ng minh r ng 3 3 sin x + sin 2 x + sin 3 x < . 2 213. [ Ngô Văn Thái ] Cho x1 , x2 ,..., xn > 0, n > 2 . Ch ng minh r ng 24
  • 25. 500 Bài Toán B t ð ng Th c Ch n L c Cao Minh Quang x12 + x2 x3 2 x2 + x3 x4 2 xn−1 + xn x1 2 xn + x1 x2 + + ... + + ≥n. x1 ( x2 + x3 ) x2 ( x3 + x4 ) xn−1 ( xn + x1 ) xn ( x1 + x2 ) 214. [ Nguy n Duy Liên ] Cho a, b, c là các s th c dương th a mãn ñi u ki n a, b, c ∈ [1, 2] . Ch ng minh r ng 1 1 1 (a + b + c) + +  ≤ 10 .     a b c 215. [ Lê Thanh H i ] Cho a, b, c d là các s th c dương. Ch ng minh r ng a2 b2 c 2 d 2 a + b + c + d + + + ≥ . b2 c2 d 2 a2 4 abcd 216. Cho x ∈ [0, 2] . Ch ng minh r ng 4 x − x3 + x + x3 ≤ 3 4 3 . 217. Cho x là m t s th c b t kì. Ch ng minh r ng 2 sin x + 15 −10 2 cos x ≤ 6 . 218. [ Tr n Văn H nh ] Cho x, y, z là các s th c dương th a mãn ñi u ki n x2 + y2 + z2 =1, n ≥ 1 . Ch ng minh r ng x y z (2n + 1) 2 n 2n +1 + + ≥ . 1 − x 2 n 1 − y 2 n 1− z 2 n 2n 219. [ Ki u Phương Chi ] Cho a, b, c là các s th c dương th a mãn ñi u ki n abc = 1 . Ch ng minh r ng 1 1 1 1 2 2 + 2 2 + 2 2 ≤ . a + 2b + 3 b + 2c + 3 c + 2a + 3 2 220. [ Vũ ð c C nh ] Cho x, y là các s th c dương th a mãn ñi u ki n x 2 + y 2 = 1 . Ch ng minh r ng  1   1  (1 + x)1 +  + (1 + y )1 +  ≥ 4 + 3 2 .         y x 221. [ Ngô Văn Thái ] Cho a, b, c ∈ (0,1] . Ch ng minh r ng 1 1 ≥ + (1− a )(1− b)(1− c) . a +b +c 3 222. [ Nguy n Văn Thông ] Cho x, y, z là các s th c dương th a mãn ñi u ki n 3x 4y 2z + + = 2. x +1 y +1 z + 1 Ch ng minh r ng 1 x3 y 4 z 2 ≤ . 89 223. [ Nguy n Bá Nam ] Cho a, b, c là các s th c dương. Ch ng minh r ng 1 1 1 3 b + c c + a a + b (a3 + b3 + c3 ) a3 + b3 + c3  ≥ 2           a + b + c  .   25
  • 26. 500 Bài Toán B t ð ng Th c Ch n L c Cao Minh Quang 224. Cho x là m t s th c b t kì. Ch ng minh r ng (16 cos4 x + 3) 4 + 768 ≥ 2048cos x . 225. [ Lê Qu c Hán ] Cho x là m t s th c b t kì. Ch ng minh r ng 8 1 (1 + x ) +16 x 4 ≤ ≤ 17 . (1+ x2 ) 4 8 226. [ Nguy n Lê Dũng ] Cho a, b, c là các s th c dương. Ch ng minh r ng a2 + b2 b2 + c2 c2 + a2 a2 + b2 + c2 + + ≤3 . a +b b+c c+a a +b+c 227. [ Tr n Xuân ðáng ] Cho a, b, c là các s th c dương, n ≥ 2 . Ch ng minh r ng a b c n n n +n +n > n −1 . b+c c+a a + b n −1 228. [ Tr nh B ng Giang ] Cho x, y, z là các s th c không âm th a ñi u ki n x + y + z = 1 , n ≥ 2 . Ch ng minh r ng nn xn y + y n z + z n x ≤ n +1 . (n + 1) 229. [ Nguy n Văn Ng c ] Cho x, y, z là các s th c dương. Ch ng minh r ng 4 4 4 16 xyz ( x + y + z ) ≤ 3 3 ( x + y ) ( y + z ) ( z + x ) . π π 230. [ Nguy n Bá ðang ] Cho x, y , z ∈  ,  . Ch ng minh r ng  6 2  sin x − sin y sin y − sin z sin z − sin x  2 1  + + ≤ 1−   . sin z sin x sin y   2  231. [ Thái Nh t Phư ng ] Cho x, y, z là các s th c dương th a mãn ñi u ki n xyz = 1 . Ch ng minh r ng x2 y2 z2 + + ≥ 3. x + y + y 3 z y + z + z 3 x z + x + x3 y 232. [ Thái Nh t Phư ng ] Cho x, y, z là các s th c dương th a mãn ñi u ki n xyz = 1 . Ch ng minh r ng x2 y 2 y2 z2 z 2 x2 + 2 2 + 2 2 ≤1 . x 2 y 2 + x7 + y 7 y z + y 7 + z 7 z x + z 7 + x7 233. [ Trương Ng c ð c ] Cho a, b, c là các s th c dương th a mãn ñi u ki n a +b +c =1. Ch ng minh r ng a b abc 3 3 + + ≤ 1+ . a + bc b + ca c + ab 4 234. [ Nguy n Minh Phương ] Cho x, y, z là các s th c dương th a mãn ñi u ki n x + y + z = 2007 . Ch ng minh r ng x 20 y 20 z 20 11 + 11 + 11 ≥ 3.6699 . y z x 26